17.01.2015 Views

Shortlisted Problems and Solutions - International Mathematical ...

Shortlisted Problems and Solutions - International Mathematical ...

Shortlisted Problems and Solutions - International Mathematical ...

SHOW MORE
SHOW LESS

You also want an ePaper? Increase the reach of your titles

YUMPU automatically turns print PDFs into web optimized ePapers that Google loves.

51 st <strong>International</strong> <strong>Mathematical</strong> Olympiad<br />

Astana, Kazakhstan 2010<br />

<strong>Shortlisted</strong> <strong>Problems</strong> with <strong>Solutions</strong>


Contents<br />

Note of Confidentiality 5<br />

Contributing Countries & Problem Selection Committee 5<br />

Algebra 7<br />

Problem A1 . . . . . . . . . . . . . . . . . . . . . . . . . . . . . . . . . . . . . . 7<br />

Problem A2 . . . . . . . . . . . . . . . . . . . . . . . . . . . . . . . . . . . . . . 8<br />

Problem A3 . . . . . . . . . . . . . . . . . . . . . . . . . . . . . . . . . . . . . . 10<br />

Problem A4 . . . . . . . . . . . . . . . . . . . . . . . . . . . . . . . . . . . . . . 12<br />

Problem A5 . . . . . . . . . . . . . . . . . . . . . . . . . . . . . . . . . . . . . . 13<br />

Problem A6 . . . . . . . . . . . . . . . . . . . . . . . . . . . . . . . . . . . . . . 15<br />

Problem A7 . . . . . . . . . . . . . . . . . . . . . . . . . . . . . . . . . . . . . . 17<br />

Problem A8 . . . . . . . . . . . . . . . . . . . . . . . . . . . . . . . . . . . . . . 19<br />

Combinatorics 23<br />

Problem C1 . . . . . . . . . . . . . . . . . . . . . . . . . . . . . . . . . . . . . . 23<br />

Problem C2 . . . . . . . . . . . . . . . . . . . . . . . . . . . . . . . . . . . . . . 26<br />

Problem C3 . . . . . . . . . . . . . . . . . . . . . . . . . . . . . . . . . . . . . . 28<br />

Problem C4 . . . . . . . . . . . . . . . . . . . . . . . . . . . . . . . . . . . . . . 30<br />

Problem C4½. . . . . . . . . . . . . . . . . . . . . . . . . . . . . . . . . . . . . 30<br />

Problem C5 . . . . . . . . . . . . . . . . . . . . . . . . . . . . . . . . . . . . . . 32<br />

Problem C6 . . . . . . . . . . . . . . . . . . . . . . . . . . . . . . . . . . . . . . 35<br />

Problem C7 . . . . . . . . . . . . . . . . . . . . . . . . . . . . . . . . . . . . . . 38<br />

Geometry 44<br />

Problem G1 . . . . . . . . . . . . . . . . . . . . . . . . . . . . . . . . . . . . . . 44<br />

Problem G2 . . . . . . . . . . . . . . . . . . . . . . . . . . . . . . . . . . . . . . 46<br />

Problem G3 . . . . . . . . . . . . . . . . . . . . . . . . . . . . . . . . . . . . . . 50<br />

Problem G4 . . . . . . . . . . . . . . . . . . . . . . . . . . . . . . . . . . . . . . 52<br />

Problem G5 . . . . . . . . . . . . . . . . . . . . . . . . . . . . . . . . . . . . . . 54<br />

Problem G6 . . . . . . . . . . . . . . . . . . . . . . . . . . . . . . . . . . . . . . 56<br />

Problem G6½. . . . . . . . . . . . . . . . . . . . . . . . . . . . . . . . . . . . . 56<br />

Problem G7 . . . . . . . . . . . . . . . . . . . . . . . . . . . . . . . . . . . . . . 60<br />

Number Theory 64<br />

Problem N1 . . . . . . . . . . . . . . . . . . . . . . . . . . . . . . . . . . . . . . 64<br />

Problem N1½. . . . . . . . . . . . . . . . . . . . . . . . . . . . . . . . . . . . . 64<br />

Problem N2 . . . . . . . . . . . . . . . . . . . . . . . . . . . . . . . . . . . . . . 66<br />

Problem N3 . . . . . . . . . . . . . . . . . . . . . . . . . . . . . . . . . . . . . . 68<br />

Problem N4 . . . . . . . . . . . . . . . . . . . . . . . . . . . . . . . . . . . . . . 70<br />

Problem N5 . . . . . . . . . . . . . . . . . . . . . . . . . . . . . . . . . . . . . . 71<br />

Problem N6 . . . . . . . . . . . . . . . . . . . . . . . . . . . . . . . . . . . . . . 72


Note of Confidentiality<br />

The <strong>Shortlisted</strong> <strong>Problems</strong> should be kept<br />

strictly confidential until IMO 2011.<br />

Contributing Countries<br />

The Organizing Committee <strong>and</strong> the Problem Selection Committee of IMO 2010 thank the<br />

following 42 countries for contributing 158 problem proposals.<br />

Armenia, Australia, Austria, Bulgaria, Canada, Columbia, Croatia,<br />

Cyprus, Estonia, Finl<strong>and</strong>, France, Georgia, Germany, Greece,<br />

Hong Kong, Hungary, India, Indonesia, Iran, Irel<strong>and</strong>, Japan,<br />

Korea (North), Korea (South), Luxembourg, Mongolia, Netherl<strong>and</strong>s,<br />

Pakistan, Panama, Pol<strong>and</strong>, Romania, Russia, Saudi Arabia, Serbia,<br />

Slovakia, Slovenia, Switzerl<strong>and</strong>, Thail<strong>and</strong>, Turkey, Ukraine,<br />

United Kingdom, United States of America, Uzbekistan<br />

Problem Selection Committee<br />

Yerzhan Baissalov<br />

Ilya Bogdanov<br />

Géza Kós<br />

Nairi Sedrakyan<br />

Damir Yeliussizov<br />

Kuat Yessenov


Algebra<br />

A1. Determine all functions f : RR such that the equality<br />

fÔÖx×yÕfÔxÕÖfÔyÕ×. (1)<br />

holds for all x, yÈR. Here, byÖx×we denote the greatest integer not exceeding x.<br />

Answer. fÔxÕconstC, where C0 or 1C2.<br />

fÔ0ÕfÔ0ÕÖfÔyÕ× (2)<br />

(France)<br />

Solution 1. First, setting x0 in (1) we get<br />

for all yÈR. Now, two cases are possible.<br />

Case 1. Assume that fÔ0Õ0. Then from (2) we conclude thatÖfÔyÕ×1 for all<br />

yÈR. Therefore, equation (1) becomes fÔÖx×yÕfÔxÕ, <strong>and</strong> substituting y0we have<br />

fÔxÕfÔ0ÕC0. Finally, fromÖfÔyÕ×1ÖC×we obtain that 1C2.<br />

Case 2. Now we have fÔ0Õ0. Here we consider two subcases.<br />

NÖz×<br />

Subcase 2a. Suppose that there exists 0α1 such that fÔαÕ0. Then setting xα<br />

in (1) we obtain 0fÔ0ÕfÔαÕÖfÔyÕ×for all yÈR. Hence,ÖfÔyÕ×0 for all yÈR. Finally,<br />

substituting x1in (1) provides fÔyÕ0for all yÈR, thus contradicting the condition<br />

fÔαÕ0.<br />

Subcase 2b. Conversely, we have fÔαÕ0for all 0α1. Consider any real z; there<br />

exists an integer N such that αz<br />

1Õ(one may set 1 if z0<strong>and</strong><br />

NÈÖ0, NÖzס1<br />

otherwise). Now, from (1) we get fÔzÕfÔÖN×αÕfÔNÕÖfÔαÕ×0 for all zÈR.<br />

Finally, a straightforward check shows that all the<br />

fÔxÕfÔÖx×aÕ<br />

obtained functions satisfy (1).<br />

Solution 2. Assume thatÖfÔyÕ×0 for some y; then the substitution x1 provides<br />

fÔyÕfÔ1ÕÖfÔyÕ×0. Hence, ifÖfÔyÕ×0for all y, then fÔyÕ0for all y. This function<br />

obviously satisfies the problem conditions.<br />

So we are left to consider the case whenÖfÔaÕ×0 for some a. Then we have<br />

fÔÖx×aÕfÔxÕÖfÔaÕ×, or<br />

This means that fÔx 1ÕfÔx 2ÕwheneverÖx fÔxÕfÔ0Õ 1×Öx 2×, hence fÔxÕfÔÖx×Õ, <strong>and</strong> we may assume<br />

that a is an integer.<br />

Now we have<br />

fÔaÕf 2a¤1<br />

2¨fÔ2aÕf<br />

1<br />

2¨fÔ2aÕÖfÔ0Õ×;<br />

this impliesÖfÔ0Õ×0, so we may even assume that a0. Therefore equation (3) provides<br />

ÖfÔ0Õ×C0<br />

for each x. Now, condition (1) becomes equivalent to the equation CCÖC×which holds<br />

exactly whenÖC×1.<br />

ÖfÔaÕ×. (3)


8<br />

A2. Let the real numbers a, b, c, d satisfy the relations a b c d6 <strong>and</strong> a 2 b 2 c 2 d 212.<br />

Prove that<br />

364Ôa 3 b 3 c 3 d 3Õ¡Ôa 4 b 4 c 4 d 4Õ48.<br />

¡ 4Õ¡ Ôd¡1Õ4¨ dÕ<br />

(Ukraine)<br />

Solution 1. Observe that<br />

4Ôa 3 b 3 c 3 d 3Õ¡Ôa 4 b 4 c 4 d Ôa¡1Õ4 6Ôa 2 b 2 c 2 d 2Õ¡4Ôa b c 4<br />

Ôa¡1Õ4 Ôb¡1Õ4 Ôc¡1Õ4 52.<br />

dÕ<br />

Now, introducing xa¡1, yb¡1, zc¡1, td¡1, we need to prove the inequalities<br />

16x 4 y 4 z 4 t 44,<br />

under the constraint<br />

x 2 y 2 z 2 t 2Ôa 2 b 2 c 2 d 2Õ¡2Ôa b c 44 (1)<br />

(we will not use the value of x y z t though it can be found).<br />

Now the rightmost inequality in (1) follows from the power 4Õ mean inequality:<br />

x 4 y 4 z 4 t 4Ôx2 y 2 z 2 t 2Õ2<br />

4.<br />

4<br />

For the other one, exp<strong>and</strong>ing the brackets we note that<br />

Ôx 2 y 2 z 2 t 2Õ2Ôx 4 y 4 z 4 t q,<br />

where q is a nonnegative number, so<br />

<strong>and</strong> we are done.<br />

x 4 y 4 z 4 t 4Ôx 2<br />

Ôb¡1Õ4 Ôc¡1Õ4 Ôd¡1Õ4¨<br />

y 2 z 2 t 2Õ216,<br />

Comment 1. The estimates are sharp; the lower <strong>and</strong> upper bounds are attained atÔ3,1,1,1Õ<strong>and</strong><br />

Ô0,2,2,2Õ, respectively.<br />

Comment 2. After the change of variables, one can finish the solution in several different ways.<br />

The latter estimate, for instance, it can be performed by moving the variables – since we need only<br />

the second of the two shifted conditions.<br />

Solution 2. First, we claim that 0a, b, c, d3. Actually, we have<br />

a b c6¡d, a 2 b 2 c 212¡d 2 ,<br />

hence the power mean inequality<br />

a 2 b 2 c 2Ôa b cÕ2<br />

3<br />

rewrites as<br />

2dÔd¡3Õ0,<br />

12¡d 2Ô6¡dÕ2<br />

3


which implies the desired inequalities for d; since the conditions are symmetric, we also have<br />

the same estimate for the other variables.<br />

Now, to prove the rightmost inequality, we use the obvious inequality x 2Ôx¡2Õ20for<br />

each real 4Õ 4Õ x; this inequality 4Õ rewrites 4Õ as 4x 3¡x44x2 . It follows 2Õ that<br />

Ô4a 3¡a 2 b 2 c 2 d 2Õ48,<br />

as desired.<br />

Now we prove the leftmost inequality in an analogous way. For each xÈÖ0, 3×, we have<br />

Ôx 1ÕÔx¡1Õ2Ôx¡3Õ0 which is equivalent to 4x 3¡x42x2 4x¡3. This implies that<br />

Ô4a 3¡a Ô4b 3¡b Ô4c 3¡c Ô4d 3¡d 4Õ2Ôa 2 b 2 c 2 d 4Ôa b c dÕ¡1236,<br />

as desired.<br />

4Õ Ô4b 3¡b<br />

4Õ Ô4c 3¡c<br />

Ô4d 3¡d 4Õ4Ôa<br />

Comment. It is easy to guess the extremal pointsÔ0,2,2,2Õ<strong>and</strong>Ô3,1,1,1Õfor this inequality. This<br />

provides a method of finding the polynomials used in Solution 2. Namely, these polynomials should<br />

have the form x 4¡4x3 ax 2 bx c; moreover, the former polynomial should have roots at 2 (with<br />

an even multiplicity) <strong>and</strong> 0, while the latter should have roots at 1 (with an even multiplicity) <strong>and</strong> 3.<br />

These conditions determine the polynomials uniquely.<br />

Solution 3. First, exp<strong>and</strong>ing 484Ôa 3 2Õ 2 b 3 2 c 3 2Õ 2Õ 2© 2Õ<br />

2 d 2Õ<strong>and</strong> applying the AM–GM inequality,<br />

we have<br />

a 4 b 4 c 4 d 4 48Ôa 4 4a Ôb 4 4b Ôc 4 4c Ôd 4 4d<br />

2¡a 4¤4a b 2 4¤4b c 2 4¤4c d 2 4¤4d<br />

4Ôa b c 3 b 3 c 3 d 3Õ,<br />

c<br />

which establishes the rightmost inequality.<br />

To prove the leftmost inequality, we first show that a, b, c, dÈÖ0, 3×as in the previous<br />

solution. Moreover, we can assume that 0abcd. Then we have a bb 2<br />

3Ôb c dÕ2<br />

3¤64.<br />

Next, we show 2Õ that 4b¡b24c¡c2 . Actually, this inequality rewrites asÔc¡bÕÔb c¡4Õ0,<br />

which follows from the previous estimate. The inequality 4a¡a24b¡b2 can be proved<br />

analogously.<br />

Further, the inequalities abctogether with 4a¡a24b¡b24c¡c2 allow us to<br />

apply the Chebyshev inequality obtaining<br />

a 2Ô4a¡a 4Õ 2Õ b 2Ô4b¡b 4Õ c<br />

2Ô4c¡c 4Õ 2Õ 2ÕÕ<br />

2Õ¨<br />

2Õ1<br />

3Ôa 2 b 2 c 4Ôa b cÕ¡Ôa 2 b 2 c<br />

Ô12¡d 2ÕÔ4Ô6¡dÕ¡Ô12¡d .<br />

3<br />

This implies that<br />

12Õ<br />

Ô4a 3¡a Ô4b 3¡b Ô4c 3¡c Ô4d 3¡d 4ÕÔ12¡d 2ÕÔd2¡4d 4d 3¡d 4<br />

3<br />

144¡48d 16d 3¡4d4<br />

4<br />

36<br />

3Ô3¡dÕÔd¡1ÕÔd 2¡3Õ. (2)<br />

3<br />

Finally, we have d 21<br />

4Ôa2 b 2 c 2 d 2Õ3(which implies d1); so, the expression<br />

3Ô3¡dÕÔd¡1ÕÔd 4<br />

2¡3Õin the right-h<strong>and</strong> part of (2) is nonnegative, <strong>and</strong> the desired inequality<br />

is proved.<br />

Comment. The rightmost inequality is easier than the leftmost one. In particular, <strong>Solutions</strong> 2 <strong>and</strong> 3<br />

show that only the condition a 2 b 2 c 2 d 212 is needed for the former one.<br />

d 3Õ4Ôa<br />

9


10<br />

A3. Let x 1 , . . .,x 100 be nonnegative real numbers such that x i x i 1 x i 21 for all<br />

i1, . . .,100 (we put x 101x 1 , x 102x 2 ). Find the maximal possible value of the sum<br />

S100<br />

x i x i 2.<br />

ôi1<br />

Answer. 25<br />

2 .<br />

1Õ 2i x 2i<br />

(Russia)<br />

Solution 1. Let x 2i0, x 2i¡11<br />

for all i1, . . ., 50. Then we have S50¤<br />

1Õ<br />

1<br />

2 2¨225<br />

. So, 2<br />

we are left to show that S25<br />

for all values of x 2 i’s satisfying the problem conditions.<br />

Consider any 1i50. By the problem condition, we get x 2i¡11¡x 2i¡x 2i 1 <strong>and</strong><br />

x 2i 21¡x 2i¡x 1Õ<br />

2i 1. Hence by the AM–GM inequality we get<br />

x 2i¡1x 2i 1 x 2i x 2i 2Ô1¡x 2i¡x 2i 1Õx 2i 1 x 2iÔ1¡x 2i¡x 2i<br />

1Õ¢Ôx Ô1¡x 2i¡x Ôx 2i x 2i 1ÕÔ1¡x 2i<br />

Å21<br />

2i¡x 2i<br />

2 4 .<br />

Summing up these inequalities for i1, 2, . . ., 50, we get the desired inequality<br />

50ôi1Ôx 2i¡1x 2i 1 x 2i x 2i 2Õ50¤1<br />

425<br />

2 .<br />

Comment. This solution shows that a bit more general fact holds. Namely, consider 2n nonnegative<br />

numbers x 1 ,... ,x 2n in a row (with no cyclic notation) <strong>and</strong> suppose that x i x i 1 x i 21 for all<br />

i1,2,... ,2n¡2. Then<br />

2n¡2 ôi1<br />

x i x i 2n¡1<br />

.<br />

4<br />

The proof is the same as above, though if might be easier to find it (for instance, applying<br />

induction). The original estimate can be obtained from this version by considering the sequence<br />

x 1 ,x 2 ,... ,x 100 ,x 1 ,x 2 .<br />

Solution 2. We present another proof of the estimate. From the problem condition, we get<br />

S100<br />

x i x i 2100 ôi1x iÔ1¡x i¡x i 1Õ100 ôi1x i¡100<br />

xi¡100<br />

ôi1 ôi1 2 x i x i 1<br />

ôi1<br />

100 100 ôi1x i¡1 ôi1Ôx i x 1Õ2 i .<br />

2<br />

By the AM–QM inequality, we haveÔx i x i 1Õ21<br />

100<br />

Ôx<br />

S100 ôi1x i¡1 ôi1Ôx i x i 1Õ«2100<br />

200£100<br />

And finally, by the AM–GM inequality<br />

i x 1Õ¨2 i<br />

x i«2<br />

ôi1<br />

ôi1 x i¡2<br />

100£100<br />

2 ôi1x i«£100<br />

100£100<br />

ôi1x i««22<br />

, so<br />

2¡100 ôi1<br />

x i«.<br />

100<br />

S2<br />

x i<br />

100¤£1<br />

2£100<br />

2¡100 ôi1<br />

100¤¢100<br />

4Å225<br />

2 .


11<br />

Comment. These solutions are not as easy as they may seem at the first sight. There are two<br />

different optimal configurations in which the variables have different values, <strong>and</strong> not all of sums of<br />

three consecutive numbers equal 1. Although it is easy to find the value 25 2<br />

, the estimates must be<br />

done with care to preserve equality in the optimal configurations.


12<br />

x<br />

¤¤¤<br />

A4. A sequence x 1 , x 2 , . . . is defined by x 11<strong>and</strong> x 2k¡x k , x 2k¡1Ô¡1Õk k1. Prove that x 1 x 2<br />

n0 for all n1.<br />

1<br />

x k for all<br />

(Austria)<br />

Solution 1. We start with some observations. First, from the definition of x i it follows that<br />

for each positive integer k we have<br />

x 4k¡3x 2k¡1¡x 4k¡2 2kx k . (1)<br />

Hence, denoting S nn<br />

i1 x i, we have<br />

S Ôx 4k¡3 x 4kkôi1 4k¡2Õ Ôx 4k¡1 x 4kÕ¨kôi1Ô0 2x kÕ2S k , (2)<br />

S 4k 2S 4k Ôx 4k 1 x 4k 2ÕS 4k . (3)<br />

Observe also that S nn<br />

i1 x in<br />

i1 1nÔmod 2Õ.<br />

Now we prove by induction on k that S i0for all i4k. The base case is valid since<br />

x 1x 3x 41, x 2¡1. For the induction step, assume that S i0for all i4k. Using<br />

the relations (1)–(3), we obtain<br />

4k 2 S 4k<br />

S 4k 42S k 10, S 4k 2S 4k0, S 4k 3S 4k 2 x 4k<br />

1<br />

4<br />

3S 0.<br />

2<br />

So, we are left to prove that S 4k 10. If k is odd, then S 4k2S k0; since k is odd, S k<br />

is odd as well, so we have S 4k2 <strong>and</strong> hence S 4k 1S 4k x 4k 11.<br />

Conversely, if k is even, then we have x 4k 1x 2k 1x k 1, hence S 4k 1S 4k x 4k<br />

2S k x k 1S k S k 10. The step is proved.<br />

<strong>and</strong> x 4k¡1x 4k¡x<br />

Solution 2. We will use the notation of S n <strong>and</strong> the relations (1)–(3) from the previous<br />

solution.<br />

Assume the contrary <strong>and</strong> consider the minimal n such that S n 10; surely n1, <strong>and</strong><br />

from S n0 we get S n0, x n 1¡1. Hence, we are especially interested in the set<br />

MØn : S n0Ù; our aim is to prove that x n 11whenever nÈM thus coming to a<br />

contradiction.<br />

For this purpose, we first describe the set M inductively. We claim that (i) M consists only<br />

of even numbers, (ii) 2ÈM, <strong>and</strong> (iii) for every even n4we have nÈMÖnß4×ÈM.<br />

Actually, (i) holds since S nnÔmod 2Õ, (ii) is straightforward, while (iii) follows from the<br />

relations S 4k 2S 4k2S k .<br />

Now, we are left to prove that x n 11if nÈM. We use the induction on n. The base<br />

case is n2, that is, the minimal element of M; here we have x 31, as desired.<br />

For the induction step, consider some 4nÈM <strong>and</strong> let mÖnß4×ÈM; then m is even,<br />

<strong>and</strong> x m 11by the induction hypothesis. We prove that x n 1x m 11. If n4m then we<br />

have x n 1x 2m 1x m 1 since m is even; otherwise, n4m 2, <strong>and</strong> x n 1¡x 2x m 1,<br />

as desired. The proof is complete.<br />

2m<br />

Comment. Using the inductive definition of set M, one can describe it explicitly. Namely, M consists<br />

exactly of all positive integers not containing digits 1 <strong>and</strong> 3 in their 4-base representation.


13<br />

A5. Denote by Q the set of all positive rational numbers. Determine all functions f : Q Q<br />

which satisfy the following equation for all x, yÈQ :<br />

f fÔxÕ2 y¨x 3<br />

fÔxyÕ. (1)<br />

(Switzerl<strong>and</strong>)<br />

fÔxÕ1<br />

Answer. The only such function is<br />

x .<br />

Solution. By substituting y1, we get<br />

fÔxÕ2¨<br />

fÔyÕ2¨<br />

f fÔxÕ2¨x 3 fÔxÕ. (2)<br />

Then, whenever fÔxÕfÔyÕ, we have<br />

x 3f f<br />

y 3<br />

which implies xy, so the function f is injective.<br />

Now replace x by xy in (2), <strong>and</strong> apply (1) twice, second time to y, fÔxÕ2¨instead ofÔx, yÕ:<br />

Since f is injective, we get<br />

f fÔxyÕ2¨ÔxyÕ3<br />

fÔxyÕy 3 f fÔxÕ2 y¨f<br />

fÔxyÕfÔxÕfÔyÕ.<br />

Therefore, f is multiplicative. This also implies fÔ1Õ1<br />

fÔxyÕ2fÔxÕ2 fÔyÕ2 ,<br />

fÔxÕ2<br />

fÔyÕ2¨.<br />

<strong>and</strong> fÔxnÕfÔxÕn for all integers n.<br />

Then the function equation (1) can be re-written as<br />

f fÔxÕ¨2 fÔxÕ<br />

fÔyÕx xfÔxÕ¨5ß2 fÔxÕ¨<br />

3 fÔxÕfÔyÕ,<br />

f fÔxÕ¨x 3 fÔxÕ. (3)<br />

Let gÔxÕxfÔxÕ. Then, by (3), we have<br />

ÐÓÓÓÓÑÓÓÓÓÒÔxÕ...¨© g gÔxÕ¨g xfÔxÕ¨xfÔxÕ¤f xfÔxÕ¨xfÔxÕ2 f<br />

xfÔxÕ2x 3 gÔxÕ¨5ß2 ,<br />

<strong>and</strong>, by induction,<br />

gÔxÕ¨Ô5ß2Õn<br />

g¡g ...g<br />

(4)<br />

n 1<br />

for every positive integer n.<br />

Consider (4) for<br />

ÐÓÓÓÓÑÓÓÓÓÒ<br />

gÔxÕ¨Ô5ß2Õn a fixed x. The left-h<strong>and</strong> side is always rational, so must be<br />

rational for every n. We show that this is possible only if gÔxÕ1. Suppose that gÔxÕ1,<br />

<strong>and</strong> let the prime factorization of gÔxÕbe gÔxÕp α 1<br />

1 . . .p α k<br />

k<br />

where p 1 , . . .,p k are distinct primes<br />

<strong>and</strong> α 1 , . . .,α k are nonzero integers. Then the unique prime factorization of (4) is<br />

g¡g ...g gÔxÕ¨Ô5ß2ÕnpÔ5ß2Õn α ¤¤¤pÔ5ß2Õn 1 α k<br />

1 k<br />

n 1<br />

ÔxÕ...¨©


14<br />

where the exponents should be integers. But this is not true for large values of n, for example<br />

. Therefore, gÔxÕ1 is impossible.<br />

yÕ y<br />

Hence, gÔxÕ1 <strong>and</strong> for all x.<br />

fÔxÕ1<br />

The function satisfies the equation (1):<br />

x fÔfÔxÕ2 1 1<br />

fÔxÕ2 x¨2 1<br />

yx3<br />

xyx 3 fÔxyÕ.<br />

fÔxÕ1<br />

Comment. Among R R functions, is not the only solution. Another solution is<br />

x<br />

f 1ÔxÕx . Using transfinite tools, infinitely many other solutions can be constructed.<br />

Ô5<br />

2Õn α 1 cannot be a integer number when 2 nЧα<br />

3ß2<br />

fÔxÕ1<br />

thus<br />

x<br />

1


gÔnÕ fÔgÔnÕÕfÔnÕ gÔfÔnÕÕ<br />

15<br />

A6. Suppose that f <strong>and</strong> g are two functions defined on the set of positive integers <strong>and</strong> taking<br />

positive integer values. Suppose also that the equations 1 <strong>and</strong><br />

1 hold for all positive integers. Prove that fÔnÕgÔnÕfor all positive integer n.<br />

(Germany)<br />

kÔxÕ¨gÔxÕ k¡1ÔxÕ¨ 1¤¤¤fÔxÕ ÐÓÓÓÓÑÓÓÓÓÒ<br />

Solution 1. Throughout the solution, by N we denote the set of all positive integers. For<br />

any function h: NN<strong>and</strong> for any positive integer k, define h kÔxÕh h . . .hÔxÕ...¨¨(in<br />

k<br />

particular, h 0ÔxÕx).<br />

Observe that f g kÔxÕ¨f g k for any positive integer k, <strong>and</strong><br />

similarly g f k. Now let a <strong>and</strong> b are the minimal values attained by f <strong>and</strong> g,<br />

respectively; say fÔn fÕa, gÔn gÕb. Then we have f g kÔn fÕ¨a k, g f kÔn gÕ¨b k, so<br />

the function f attains all values from the set N fØa, a 1, . . .Ù, while g attains all the values<br />

from the set N gØb, b 1, . . .Ù.<br />

Next, note that fÔxÕfÔyÕimplies gÔxÕg fÔxÕ¨¡1g fÔyÕ¨¡1gÔyÕ; surely, the<br />

converse implication also holds. Now, we say that x <strong>and</strong> y are similar (<strong>and</strong> write xy) if<br />

fÔxÕfÔyÕ(equivalently, gÔxÕgÔyÕ). For every xÈN, we defineÖx×ØyÈN:xyÙ;<br />

surely, y 1y 2 for all y 1 , y 2ÈÖx×, soÖx×Öy×whenever yÈÖx×.<br />

Ð<br />

Now we investigate the structure of the setsÖx×.<br />

Claim 1. Suppose that fÔxÕfÔyÕ; then xy, that is, fÔxÕfÔyÕ. Consequently, each<br />

classÖx×contains at most one element from N f , as well as at most one element from N g .<br />

Proof. If fÔxÕfÔyÕ, then we have gÔxÕg fÔxÕ¨¡1g fÔyÕ¨¡1gÔyÕ, so xy. The<br />

second statement follows now from the sets of values of f <strong>and</strong> g.<br />

Next, we clarify which classes do not contain large elements.<br />

fÔyÕ<br />

Claim 2. For any xÈN, we haveÖx×Ø1,<br />

gÔyÕ¨fÔyÕ gÔyÕ¨<br />

2, . . .,b¡1Ùif <strong>and</strong> only if fÔxÕa. Analogously,<br />

Ð<br />

Öx×Ø1, 2, . . ., a¡1Ùif <strong>and</strong> only if gÔxÕb.<br />

Proof. We will prove thatÖx×Ø1, 2, . . .,b¡1ÙfÔxÕa; the proof of the second<br />

statement is similar.<br />

Note that fÔxÕaimplies that there exists some y satisfying fÔyÕfÔxÕ¡1, so f<br />

1fÔxÕ, <strong>and</strong> hence xgÔyÕb. Conversely, if bcxthen cgÔyÕfor some yÈN,<br />

which in turn follows fÔxÕf 1a 1, <strong>and</strong> hence fÔxÕa.<br />

Claim 2 implies that there exists exactly one class contained inØ1, . . .,a¡1Ù(that is, the<br />

classÖn g×), as well as exactly one class contained inØ1, . . .,b¡1Ù(the classÖn f×). Assume for a<br />

moment that ab; thenÖn g×is contained inØ1, . . .,b¡1Ùas well, hence it coincides withÖn g×.<br />

So, we get that<br />

Ð<br />

Claim 3. ab.<br />

Proof. By Claim 2, we haveÖa×Ön f×, soÖa×should contain some element a½b by Claim 2<br />

again. If aa½, thenÖa×contains two elementsawhich is impossible by Claim 1. Therefore,<br />

aa½b. Similarly, ba.<br />

Now we are ready to prove the problem statement. First, we establish the following<br />

fÕ Ð<br />

d.<br />

Proof. Induction on d. For d0, the statement follows from (1) <strong>and</strong> Claim 3. Next, for d1<br />

f g dÔn fÕ¨fÔn da d.<br />

d is analogous.<br />

fÔxÕagÔxÕbxn fn g . (1)<br />

Claim 4. For every integer d0, f d 1Ôn fÕgd 1Ôn fÕa from the induction hypothesis we have f d 1Ôn fÕf dÔn fÕ¨f The equality g d 1Ôn fÕa


16<br />

Finally, for each xÈN, we have fÔxÕa d for some d0, so fÔxÕf g dÔn fÕ¨<strong>and</strong><br />

hence xgdÔn fÕ. It follows that gÔxÕg g dÔn fÕ¨gd 1Ôn fÕa fÔaÕ¨gÔaÕ<br />

dfÔxÕby Claim 4.<br />

Solution 2. We start with the same observations, introducing the relation<strong>and</strong> proving<br />

Claim<br />

fÔaÕfÔxÕ<br />

1 from the previous solution.<br />

Note that fÔaÕa since otherwise we have fÔaÕa<strong>and</strong> hence gÔaÕg Ð1,<br />

which is false.<br />

Claim 2½. ab.<br />

Proof. We can assume that ab. Since fÔaÕa 1, there exists some xÈNsuch that<br />

1, which is equivalent to fÔaÕf gÔxÕ¨<strong>and</strong> agÔxÕ. fÔaÕfÔxÕ<br />

Since gÔxÕba, by<br />

Claim 1 we have agÔxÕb, which together with ab proves the Claim.<br />

Now, almost the same method allows to find the values fÔaÕ<strong>and</strong> gÔaÕ.<br />

Claim 3½. fÔaÕgÔaÕa 1.<br />

Proof. Assume the contrary; then fÔaÕa 2, hence there exist some x, yÈNsuch that<br />

fÔxÕfÔaÕ¡2 <strong>and</strong> fÔyÕgÔxÕ(as gÔxÕab). Now we get 2f g 2ÔxÕ¨,<br />

so ag2ÔxÕa, <strong>and</strong> by Claim<br />

gÔxÕ¨fÔxÕ<br />

1 we get ag2ÔxÕg 1Õ<br />

fÔyÕ¨1 gÔyÕ1 a; this is<br />

impossible. The equality gÔaÕa 1 is similar.<br />

Now, we are prepared for the proof of the problem statement. First, we prove it for na.<br />

Claim 4½. For each integer xa, we have fÔxÕgÔxÕx 1.<br />

Proof. Induction on x. The base case xais provided by Claim 3½, while the induction<br />

step<br />

fÔnÕ gÔnÕ¨gÔnÕ fÔnÕ fÔfÔnÕÕ 1<br />

follows from fÔx 1Õf 1Ôx 1 <strong>and</strong> the similar computation<br />

for gÔx 1Õ.<br />

Finally, for an arbitrary nÈNwe have gÔnÕa, so by Claim 4½we have<br />

f 1, hence fÔnÕgÔnÕ.<br />

Comment. It is not hard now to describe all the functions f : NN satisfying the property<br />

1. For each such function, there exists n 0ÈN such that fÔnÕn 1 for all nn 0 , while for<br />

each nn 0 , fÔnÕis an arbitrary number greater than of equal to n 0 (these numbers may be different<br />

for different nn 0 ).


17<br />

A7. Let a 1 , . . .,a r be positive real numbers. For nr, we inductively define<br />

a nmax<br />

1kn¡1Ôa k a n¡kÕ. (1)<br />

Prove that there exist positive integers lr <strong>and</strong> N such that a na n¡l a l for all nN.<br />

¤¤¤<br />

(Iran)<br />

Solution 1. First, from the problem conditions we have that each a n (nr) can be expressed<br />

as a na j1 a j2 with j 1 , j 2n, j 1 j 2n. If, say, j 1r then we can proceed in the same<br />

way with a j1 , <strong>and</strong> so on. Finally, we represent a n in a form<br />

a na i1 a ik , (2)<br />

i kn. (3)<br />

Moreover, if a i1<br />

¤¤¤<br />

<strong>and</strong> a i2 are the numbers in (2) obtained on the last step, then i 1 i 2r.<br />

Hence we can adjust (3) as<br />

i kn, i 1 i 2r.<br />

¤¤¤<br />

(4)<br />

On the other h<strong>and</strong>, suppose that the indices i 1 , . . .,i k satisfy the conditions (4). Then,<br />

denoting s ji 1 i j , from (1) we have<br />

a na ska sk¡1 ¤¤¤<br />

a ika sk¡2 a ik¡1 a ik¤¤¤a i1 a ik<br />

Ð<br />

.<br />

Summarizing these observations we get the following<br />

Claim. For every nr, we have<br />

a nmaxØa i1 a ik : the collectionÔi 1 , . . .,i kÕsatisfies (4)Ù.<br />

¤¤¤<br />

Now we denote<br />

a i<br />

smax<br />

1ir i<br />

l<br />

<strong>and</strong> fix some index lr such that sa l .<br />

Consider some nr2 l 2r <strong>and</strong> choose an expansion of a n in the form (2), (4). Then we have<br />

ni 1 i krk,<br />

¤¤¤ ¤¤¤<br />

so knßrrl 2. Suppose that none of the numbers i 3 , . . ., i k equals l.<br />

Then by the pigeonhole principle there is an index 1jr which appears among i 3 , . . .,i k<br />

at least l times, <strong>and</strong> surely jl. Let us delete these l occurrences of j fromÔi 1 , . . ., i kÕ, <strong>and</strong><br />

add j occurrences of l instead, obtaining a sequenceÔi 1 , i 2 , i½3 , . . .,i½k½Õalso satisfying (4). By<br />

Claim, we have<br />

a i1 a ika na i1 a i2 a i½3<br />

a i½k½,<br />

or, after removing the coinciding terms, la jja¤¤¤ l , so a l j<br />

. By the definition of l, this<br />

la j<br />

means that la jja l , hence<br />

a na i1 a i2<br />

¤¤¤<br />

a i½3<br />

a i½k½.<br />

Thus, for every nr2 l 2r we have found a representation of the form (2), (4) with i jl<br />

for some j3. Rearranging the indices we may assume<br />

ik¡1Õ<br />

that i kl.<br />

Finally, observe that in this representation, the indicesÔi 1 , . . ., i k¡1Õsatisfy the conditions<br />

(4) with n replaced by n¡l. Thus, from the Claim we get<br />

a n¡l a lÔa i1 a a la n ,<br />

which by (1) implies<br />

a na n¡l a l for each nr 2 l 2r,<br />

as desired.<br />

1i jr, i 1 ¤¤¤<br />

1i jr, i 1 ¤¤¤


18<br />

Solution 2. As in the previous solution, we involve the expansion (2), (3), <strong>and</strong> we fix some<br />

index 1lr such that<br />

a l a i<br />

lsmax<br />

1ir i .<br />

Now, we introduce the sequenceÔb nÕas b na n¡sn; then b l0.<br />

We prove by induction on n that b n0, <strong>and</strong>Ôb nÕsatisfies the same recurrence relation<br />

asÔa nÕ. The base cases nrfollow from the definition of s. Now, for nrfrom the<br />

induction hypothesis we have<br />

b nmax<br />

1kn¡1Ôa k a n¡kÕ¡nsmax<br />

1kn¡1Ôb k b n¡k nsÕ¡nsmax<br />

1kn¡1Ôb k b n¡kÕ0,<br />

as required.<br />

Now, if b k0 for all 1kr, then b n0 for all n, hence a nsn, <strong>and</strong> the statement is<br />

trivial. Otherwise, define<br />

Mmax<br />

1irb i,<br />

εminØb i:1ir, b i0Ù.<br />

Then for nr we obtain<br />

b nmax<br />

1kn¡1Ôb k b n¡kÕb<br />

l<br />

b n¡lb n¡l,<br />

so<br />

¤¤¤<br />

contained in a set<br />

b<br />

We claim that this set is finite. Actually, for any xÈT, let xb i1 b ik (i 1 , . . .,i kr).<br />

Then among b ij ’s there are at most M nonzero terms (otherwise xM<br />

Thus<br />

ε ε¤Ô¡εÕ¡M).<br />

x can be expressed in the same way with kM<br />

, <strong>and</strong> there is only a finite number of such<br />

ε<br />

sums.<br />

0b nb ¤¤¤<br />

n¡lb n¡2l¤¤¤¡M.<br />

Thus, in view of the expansion (2), (3) applied to the sequenceÔb nÕ, 0×<br />

we get that each b n is<br />

TØb i1 b i2 ik : i 1 , . . .,i krÙÖ¡M,<br />

Finally, for every t1, 2, . . ., l we get that the sequence<br />

b r t, b r t l, b r t 2l, . . .<br />

is non-decreasing <strong>and</strong> attains the finite number of values; therefore it is constant from some<br />

index. Thus, the sequenceÔb nÕis periodic with period l from some index N, which means that<br />

b nb Ôn¡lÕsÕ<br />

n¡lb n¡l b l for all nN l,<br />

<strong>and</strong> hence<br />

a nb n nsÔb n¡l Ôb l lsÕa n¡l a l for all nN l,<br />

as desired.


TÕ<br />

19<br />

A8. Given six positive numbers a, b, c, d, e, f such that abcdef. Let a c eS<br />

<strong>and</strong> b d fT. Prove that<br />

2ST3ÔS SÔbd bf dfÕ TÔac ae ceÕ¨. (1)<br />

fÕÔx¡aÕÔx¡cÕÔx¡eÕ eÕÔx¡bÕÔx¡dÕÔx¡fÕ<br />

(South Korea)<br />

Solution 1. We define also σac ce ae, τbd bf df. The idea of the solution is to<br />

interpret (1) as a natural inequality on the roots of an appropriate polynomial.<br />

Actually, consider the polynomial<br />

PÔxÕÔb d Ôa c<br />

σx¡aceÕ TÔx 3¡Sx 2 SÔx 3¡Tx 2 τx¡bdfÕ. (2)<br />

Surely, P is cubic with leading coefficient S T0. Moreover, we have<br />

PÔaÕSÔa¡bÕÔa¡dÕÔa¡fÕ0,<br />

PÔeÕSÔe¡bÕÔe¡dÕÔe¡fÕ0,<br />

PÔcÕSÔc¡bÕÔc¡dÕÔc¡fÕ0,<br />

PÔfÕTÔf¡aÕÔf¡cÕÔf¡eÕ0.<br />

Hence, each of the intervalsÔa, cÕ,Ôc, eÕ,Ôe, fÕcontains at least one root of PÔxÕ. Since there<br />

are at most three roots at all, we obtain that there is exactly one root in each interval (denote<br />

them by αÈÔa, cÕ, βÈÔc, eÕ, γÈÔe, fÕ). Moreover, the polynomial P can be factorized as<br />

PÔxÕÔT SÕÔx¡αÕÔx¡βÕÔx¡γÕ. (3)<br />

Equating the coefficients in the two representations (2) <strong>and</strong> (3) of PÔxÕprovides<br />

α β γ2TS<br />

T S ,<br />

αβ αγ βγSτ Tσ<br />

T S .<br />

Now, since the numbers α, β, γ are distinct, we have<br />

0Ôα¡βÕ2 which implies<br />

β<br />

or<br />

4S 2 T 23ÔT SÕÔTσ SτÕ,<br />

4S 2 T 2<br />

ÔT SÕ2Ôα<br />

Ôα¡γÕ2 Ôβ¡γÕ22Ôα β γÕ2¡6Ôαβ αγ βγÕ,<br />

TσÕ<br />

γÕ23Ôαβ αγ βγÕ3ÔSτ<br />

,<br />

T S<br />

which is exactly what we need.<br />

Comment 1. In fact, one can locate the roots of PÔxÕmore narrowly: they should lie in the intervals<br />

Ôa,bÕ,Ôc,dÕ,Ôe,fÕ.<br />

Surely, if we change all inequality signs<br />

fÕ<br />

in the problem statement to non-strict ones, the (non-strict)<br />

inequality will also hold by continuity. One can also find when the equality is achieved. This happens<br />

in that case when PÔxÕis a perfect cube, which immediately implies that bcdeÔαβγÕ,<br />

together with the additional condition that P¾ÔbÕ0. Algebraically,<br />

6ÔT SÕb¡4TS0<br />

3bÔa fÕ2Ôa 2bÕÔ2b<br />

fbÔ4b¡aÕ 3Ôb¡aÕ<br />

2a bb¢1<br />

2a bÅb.<br />

This means that for every pair of numbers a, b such that 0ab, there exists fb such that the<br />

pointÔa,b,b,b,b,fÕis a point of equality.


20<br />

Solution 2. Let<br />

U1<br />

Ôe¡aÕ2 Ôc¡aÕ2 Ôe¡cÕ2¨S 2¡3Ôac ae ceÕ<br />

2<br />

TÕ<br />

<strong>and</strong><br />

2¡VÕ TÕ<br />

V1<br />

Ôf¡bÕ2 Ôf¡dÕ2 Ôd¡bÕ2¨T 2¡3Ôbd bf dfÕ.<br />

2<br />

ThenÔL.H.S.Õ2¡ÔR.H.S.Õ2Ô2STÕ2¡ÔS S¤3Ôbd bf dfÕ ceÕ¨<br />

T¤3Ôac ae<br />

4S 2 T 2¡ÔS SÔT SVÕ<br />

TÔS 2¡UÕ¨ÔS TÕÔSV TUÕ¡STÔT¡SÕ2 ,<br />

<strong>and</strong> the statement is equivalent with<br />

ÔS TÕÔSV TUÕSTÔT¡SÕ2 . (4)<br />

By the Cauchy-Schwarz inequality,<br />

ÔS TÕÔTU<br />

S¤TU<br />

T¤SV¨2ST<br />

U V¨2<br />

Ôe¡aÕ Ôc¡aÕ Ôf¡bÕ Ôf¡dÕ 2Å<br />

. (5)<br />

Estimate the quantitiesU <strong>and</strong>V by the QM–AM inequality with the positive termsÔe¡cÕ2<br />

<strong>and</strong>Ôd¡bÕ2 being omitted:<br />

ÔT¡SÕ 2Ôe¡dÕ 2¡a© Ôf¡bÕ2 U<br />

VÔe¡aÕ2 Ôc¡aÕ2<br />

Ôf¡dÕ2<br />

2 2<br />

¡e c<br />

2 2¡b<br />

2<br />

Ôc¡bÕÔc¡dÕ<br />

3<br />

Ôe¡fÕÔe¡dÕ<br />

3<br />

(6)<br />

2Ôc¡bÕT¡S.<br />

The estimates (5) <strong>and</strong> (6) prove (4) <strong>and</strong> hence the statement.<br />

Solution 3. We keep using the notations σ <strong>and</strong> τ from Solution 1. Moreover, let sc e.<br />

Note that<br />

Ôe¡fÕÔc¡bÕ0,<br />

since each summ<strong>and</strong> is negative. This rewrites as<br />

Ôbd bf dfÕ¡Ôac ce aeÕÔc eÕÔb d f¡a¡c¡eÕ, or<br />

τ¡σsÔT¡SÕ. (7)<br />

Then we have<br />

Sτ TσSÔτ¡σÕ<br />

SsÔT¡SÕ<br />

TÕσSsÔT¡SÕ ÔS ÔS TÕÔce asÕ<br />

TÕsÅ<br />

TÕ¢s 2<br />

ÔS¡sÕsÅs¢2ST¡3<br />

ÔS TÕsÅ.<br />

4 4ÔS Using this inequality together with the AM–GM inequality we get<br />

3<br />

TσÕ3<br />

TÕÔSτ TÕs¢2ST¡3<br />

4ÔS TÕ<br />

4ÔS 4ÔS 3<br />

4ÔS TÕs 2ST¡3<br />

4ÔS TÕs<br />

ST.<br />

2<br />

Hence,<br />

2ST3ÔS SÔbd bf dfÕ TÔac ae ceÕ¨.<br />

2<br />

¢f¡d


Comment 2. The expression (7) can be found by considering the sum of the roots of the quadratic<br />

polynomial qÔxÕÔx¡bÕÔx¡dÕÔx¡fÕ¡Ôx¡aÕÔx¡cÕÔx¡eÕ.<br />

Solution 4. We introduce the expressions σ <strong>and</strong> τ as in the previous solutions. The idea of<br />

the solution is to change the values of variables a, . . .,f keeping the left-h<strong>and</strong> side unchanged<br />

<strong>and</strong> increasing the right-h<strong>and</strong> side; it will lead to a simpler inequality which can be proved in<br />

a direct way.<br />

abcd<br />

Namely, we change the variables (i) keeping the (non-strict) inequalities<br />

x½<br />

ef; (ii) keeping the values of sums S <strong>and</strong> T unchanged; <strong>and</strong> finally (iii) increasing<br />

zÕ<br />

the<br />

values of σ <strong>and</strong> τ. Then the left-h<strong>and</strong> side of (1) remains unchanged, while the right-h<strong>and</strong><br />

side increases. Hence, the inequality (1) (<strong>and</strong> even a non-strict version of (1)) for the changed<br />

values would imply the same (strict) inequality for the original values.<br />

First, we find<br />

zÕzÔx½<br />

the sufficient conditions for (ii) <strong>and</strong> (iii) to be satisfied.<br />

y½Õ x½y½zÔx½<br />

Lemma. Let x, y, z0; denote UÔx, y, zÕx y z, υÔx, y, zÕxy xz yz. Suppose that<br />

y½x y butx¡yx½¡y½; then we<br />

yÕ y½Õ Ôx½<br />

have UÔx½, y½, zÕUÔx, y, zÕ<strong>and</strong> υÔx½, y½,<br />

υÔx, y, zÕwith equality achieved only whenx¡yx½¡y½.<br />

Proof. The first equality is obvious. For the second, we have<br />

y½Õ2¡Ôx½¡y½Õ2<br />

υÔx½, y½,<br />

4<br />

Ôx yÕ2¡Ôx¡yÕ2<br />

zÔx υÔx, y, zÕ,<br />

4<br />

with the equality achieved only forÔx½¡y½Õ2Ôx¡yÕ2x½¡y½x¡y, as desired.<br />

ÐNow, we apply Lemma several times making the following changes. For each change, we<br />

denote the new values by the same letters to avoid cumbersome notations.<br />

1. Let kd¡c<br />

2 . ReplaceÔb, c, d, eÕbyÔb k, c k, d¡k, e¡kÕ. After the change we have<br />

abcdef, the values of S, T remain unchanged, but σ, τ strictly increase by<br />

Lemma.<br />

2. Let le¡d<br />

2 . ReplaceÔc, d, e, fÕbyÔc l, d l, e¡l, f¡lÕ. After the change we<br />

dÕ<br />

have<br />

abcdef, the values of S, T remain unchanged, but σ, τ strictly increase by the<br />

Lemma.<br />

3. Finally, let mc¡b<br />

3 . ReplaceÔa, b, c, d, e, fÕbyÔa 2m, b 2m, c¡m, d¡m, e¡m, f¡mÕ.<br />

After the change, we have abcdef <strong>and</strong> S, T are unchanged. To check (iii),<br />

we observe that our change can be considered as a composition of two changes:Ôa, b, c,<br />

Ôa m, b m, c¡m, d¡mÕ<strong>and</strong>Ôa, b, e, fÕÔa fÕ<br />

m, b m, e¡m, f¡mÕ. It is easy to see that<br />

each of these two consecutive changes satisfy the conditions of the Lemma, hence the values<br />

of σ <strong>and</strong> τ increase.<br />

Finally, we come to the situation when abcdef, <strong>and</strong> we need to prove the<br />

inequality<br />

2Ôa 2bÕÔ2b 2bfÕ 2Õ¨<br />

fÕ3Ôa Ô2b fÕÔ2ab b<br />

Now, observe that<br />

fÕ¤<br />

4b<br />

3bÔa 4b<br />

2¤2Ôa 2bÕÔ2b fÕ3bÔa 4b fÕ<br />

Ôa 2bÕÔb<br />

2 2bÕÔb<br />

Ôa<br />

Ôa 2bÕÔb<br />

2fÕ<br />

2fÕ<br />

21<br />

Ô2b fÕÔ2a bÕ¨. (8)<br />

Ô2a bÕÔ2b fÕ¨.


22<br />

HenceÔ4Õrewrites as<br />

3bÔa 4b fÕ<br />

2fÕ<br />

fÕ¤ 23bÔa 4b<br />

Ôa 2bÕÔb<br />

which is simply the AM–GM inequality.<br />

fÕ¨<br />

Ô2a bÕÔ2b<br />

2fÕ<br />

Ôa 2bÕÔb<br />

Ô2b fÕÔ2a bÕ¨,<br />

Comment 3. Here, we also can find all the cases of equality. Actually, it is easy to see that if<br />

some two numbers among b,c,d,e are distinct then one can use Lemma to increase the right-h<strong>and</strong> side<br />

of (1). Further, if bcde, then we need equality inÔ4Õ; this means that we apply AM–GM to<br />

equal numbers, that is,<br />

3bÔa 4b 2fÕ fÕÔa 2bÕÔb Ô2a bÕÔ2b fÕ,<br />

which leads to the same equality as in Comment 1.


Combinatorics<br />

C1. In a concert, 20 singers will perform. For each singer, there is a (possibly empty) set of<br />

other singers such that he wishes to perform later than all the singers from that set. Can it<br />

happen that there are exactly 2010 orders of the singers such that all their wishes are satisfied<br />

Answer. Yes, such an example exists.<br />

(Austria)<br />

Solution. We say that an order of singers is good if it satisfied all their wishes. Next, we<br />

say that a number N is realizable by k singers (or k-realizable) if for some set of wishes of<br />

these singers there are exactly N good orders. Thus, we have to prove that a number 2010 is<br />

20-realizable.<br />

We start with the following simple<br />

Lemma. Suppose that numbers n 1 , n 2 are realizable by respectively k 1 <strong>and</strong> k 2 singers. Then<br />

the number n 1 n 2 isÔk 1 k 2Õ-realizable.<br />

Proof. Let the singers A 1 , . . ., A k1 (with some wishes among them) realize n 1 , <strong>and</strong> the singers B 1 ,<br />

..., B k2 (with some wishes among them) realize n 2 . Add to each singer B i the wish to perform<br />

later than all the singers A j . Then, each good order of the obtained set of singers has the form<br />

ÔA i1 , . . .,A ik1 , B j1 , . . .,B jk2Õ, whereÔA i1 , . . .,A ik1Õis a good order for A i ’s <strong>and</strong>ÔB j1 , . . .,B jk2Õ<br />

Ð<br />

is a good order for B j ’s. Conversely, each order of this form is obviously good. Hence, the<br />

number of good orders is n 1 n 2 .<br />

In view of Lemma, we show how to construct sets of singers containing 4, 3 <strong>and</strong> 13 singers<br />

<strong>and</strong> realizing the numbers 5, 6 <strong>and</strong> 67, respectively. Thus the number 20106¤5¤67 will be<br />

realizable by 4 3 1320 singers. These companies of singers are shown in Figs. 1–3; the<br />

wishes are denoted by arrows, <strong>and</strong> the number of good orders for each Figure st<strong>and</strong>s below in<br />

the brackets.<br />

a 3<br />

a 2<br />

a 1<br />

a<br />

b<br />

a 4<br />

c<br />

(5)<br />

Fig. 1<br />

d<br />

(3)<br />

Fig. 2<br />

y<br />

a 5<br />

a 6<br />

a 7<br />

a 8<br />

a 9<br />

a 10<br />

x<br />

a 11<br />

(67)<br />

Fig. 3<br />

For Fig. 1, there are exactly 5 good ordersÔa, b, c, dÕ,Ôa, b, d, cÕ,Ôb, a, c, dÕ,Ôb, a, d, cÕ,<br />

Ôb, d, a, cÕ. For Fig. 2, each of 6 orders is good since there are no wishes.


24<br />

Finally, for Fig. 3, the order of a 1 , . . .,a 11 is fixed; in this line, singer x can st<strong>and</strong> before<br />

each of a i (i9), <strong>and</strong> singer y can st<strong>and</strong> after each of a j (j5), thus resulting in 9¤763<br />

cases. Further, the positions of x <strong>and</strong> y in this line determine the whole order uniquely unless<br />

both of them come between the same pairÔa i , a i 1Õ(thus 5i8); in the latter cases, there<br />

are two orders instead of 1 due to the order of x <strong>and</strong> y. Hence, the total number of good orders<br />

is 63 467, as desired.<br />

Comment. The number 20 in the problem statement is not sharp <strong>and</strong> is put there to respect the<br />

original formulation. So, if necessary, the difficulty level of this problem may be adjusted by replacing<br />

20 by a smaller number. Here we present some improvements of the example leading to a smaller<br />

number of singers.<br />

Surely, each example with20 singers can be filled with some “super-stars” who should perform<br />

at the very end in a fixed order. Hence each of these improvements provides a different solution for<br />

the problem. Moreover, the large variety of ideas st<strong>and</strong>ing behind these examples allows to suggest<br />

that there are many other examples.<br />

1. Instead of building the examples realizing 5 <strong>and</strong> 6, it is more economic to make an example<br />

realizing 30; it may seem even simpler. Two possible examples consisting of 5 <strong>and</strong> 6 singers are shown<br />

in Fig. 4; hence the number 20 can be decreased to 19 or 18.<br />

For Fig. 4a, the order of a 1 ,... ,a 4 is fixed, there are 5 ways to add x into this order, <strong>and</strong> there<br />

are 6 ways to add y into the resulting order of a 1 ,...,a 4 ,x. Hence there are 5¤630 good orders.<br />

On Fig. 4b, for 5 singers a,b 1 ,b 2 ,c 1 ,c 2 there are 5!120 orders at all. Obviously, exactly one half<br />

of them satisfies the wish b 1b 2 , <strong>and</strong> exactly one half of these orders satisfies another wish c 1c 2 ;<br />

hence, there are exactly 5!ß430 good orders.<br />

a 1<br />

a 2<br />

a 3<br />

x<br />

y<br />

b 1<br />

a<br />

c 2<br />

b 2<br />

c 1<br />

y<br />

b 2 b<br />

b 3 1<br />

b 4 b 5<br />

a 6<br />

a7<br />

a8<br />

a 9<br />

x<br />

b 1 b 2<br />

b 3 b 4<br />

a 5<br />

y a 6<br />

a7 x<br />

a8<br />

c 9 c 10<br />

a 4<br />

(30)<br />

a)<br />

(30)<br />

b)<br />

a 11<br />

(2010)<br />

a 10<br />

(2010)<br />

Fig. 4 Fig. 5 Fig. 6<br />

c 11<br />

2. One can merge the examples for 30 <strong>and</strong> 67 shown in Figs. 4b <strong>and</strong> 3 in a smarter way, obtaining<br />

a set of 13 singers representing 2010. This example is shown in Fig. 5; an arrow from/to group<br />

Øb 1 ,...,b 5Ùmeans that there exists such arrow from each member of this group.<br />

Here, as in Fig. 4b, one can see that there are exactly 30 orders of b 1 ,... ,b 5 ,a 6 ,...,a 11 satisfying<br />

all their wishes among themselves. Moreover, one can prove in the same way as for Fig. 3 that each<br />

of these orders can be complemented by x <strong>and</strong> y in exactly 67 ways, hence obtaining 30¤672010<br />

good orders at all.<br />

Analogously, one can merge the examples in Figs. 1–3 to represent 2010 by 13 singers as is shown<br />

in Fig. 6.


25<br />

a 5<br />

a 1<br />

a 2<br />

a 3<br />

b 5<br />

b 6<br />

(67)<br />

b 4<br />

a 4<br />

b 1<br />

b 2<br />

b 3<br />

a 4<br />

b 3<br />

b 2<br />

b 4<br />

a 3<br />

a<br />

a 6<br />

2<br />

a 1<br />

b 1<br />

(2010)<br />

Fig. 7 Fig. 8<br />

3. Finally, we will present two other improvements; the proofs are left to the reader. The graph in<br />

Fig. 7 shows how 10 singers can represent 67. Moreover, even a company of 10 singers representing 2010<br />

can be found; this company is shown in Fig. 8.


26<br />

C2. On some planet, there are 2 N countries (N4). Each country has a flag N units wide<br />

<strong>and</strong> one unit high composed of N fields of size 1¢1, each field being either yellow or blue. No<br />

two countries have the same flag.<br />

We say that a set of N flags is diverse if these flags can be arranged into an N¢N square so<br />

that all N fields on its main diagonal will have the same color. Determine the smallest positive<br />

integer M such that among any M distinct flags, there exist N flags forming a diverse set.<br />

Answer. M2 N¡2 1.<br />

(Croatia)<br />

Solution. When speaking about the diagonal of a square, we will always mean the main<br />

diagonal.<br />

Let M N be the smallest positive integer satisfying the problem condition. First, we show<br />

that M N2N¡2 . Consider the collection of all 2 N¡2 flags having yellow first squares <strong>and</strong> blue<br />

second ones. Obviously, both colors appear on the diagonal of each N¢N square formed by<br />

these flags.<br />

We are left to show that M N2N¡2 1, thus obtaining the desired answer. We start with<br />

establishing this statement for N4.<br />

Suppose that we have 5 flags of length 4. We decompose each flag into two parts of 2 squares<br />

each; thereby, we denote each flag as LR, where the 2¢1 flags L, RÈSØBB, BY, YB, YYÙ<br />

are its left <strong>and</strong> right parts, respectively. First, we make two easy observations on the flags 2¢1<br />

which can be checked manually.<br />

(i) For each AÈS, there exists only one 2¢1 flag CÈS (possibly CA) such that A<br />

<strong>and</strong> C cannot form a 2¢2 square with monochrome diagonal (for part BB, that is YY, <strong>and</strong><br />

for BY that is YB).<br />

(ii) Let A 1 , A 2 , A 3ÈS be three distinct elements; then two of them can form a 2¢2 square<br />

with yellow diagonal, <strong>and</strong> two of them can form a 2¢2 square with blue diagonal (for all parts<br />

but BB, a pair (BY, YB) fits for both statements, while for all parts but BY, these pairs are<br />

(YB, YY) <strong>and</strong> (BB, YB)).<br />

Now, let l <strong>and</strong> r be the numbers of distinct left <strong>and</strong> right parts of our 5 flags, respectively.<br />

The total number of flags is 5rl, hence one of the factors (say, r) should be at least 3. On<br />

the other h<strong>and</strong>, l, r4, so there are two flags with coinciding right part; let them be L 1 R 1<br />

<strong>and</strong> L 2 R 1 (L 1L 2 ).<br />

Next, since r3, there exist some flags L 3 R 3 <strong>and</strong> L 4 R 4 such that R 1 , R 3 , R 4 are distinct.<br />

Let L½R½be the remaining flag. By (i), one of the pairsÔL½, L 1Õ<strong>and</strong>ÔL½, L 2Õcan form a<br />

2¢2 square with monochrome diagonal; we can assume that L½, L 2 form a square with a blue<br />

diagonal. Finally, the right parts of two of the flags L 1 R 1 , L 3 R 3 , L 4 R 4 can also form a 2¢2<br />

square with a blue diagonal by (ii). Putting these 2¢2 squares on the diagonal of a 4¢4<br />

square, we find a desired arrangement of four flags.<br />

We are ready to prove the problem statement by induction on N; actually, above we have<br />

proved the base case N4. For the induction step, assume that N4, consider any 2 N¡2 1<br />

flags of length N, <strong>and</strong> arrange them into a large flag of sizeÔ2N¡2 1Õ¢N. This flag contains<br />

a non-monochrome column since the flags are distinct; we may assume that thisÎ2 column is the<br />

first one. By the pigeonhole principle, this column contains at leastÊ2 N¡2 1 N¡3 1<br />

2<br />

squares of one color (say, blue). We call the flags with a blue first square good.<br />

Consider all the good flags <strong>and</strong> remove the first square from each of them. We obtain at<br />

least 2 N¡3 1M N¡1 flags of length N¡1; by the induction hypothesis, N¡1 of them


27<br />

can form a square Q with the monochrome diagonal. Now, returning the removed squares, we<br />

obtain a rectangleÔN¡1Õ¢N, <strong>and</strong> our aim is to supplement it on the top by one more flag.<br />

If Q has a yellow diagonal, then we can take each flag with a yellow first square (it exists<br />

by a choice of the first column; moreover, it is not used in Q). Conversely, if the diagonal of Q<br />

is blue then we can take any of the2 N¡3 1¡ÔN¡1Õ0remaining good flags. So, in both<br />

cases we get a desired N¢N square.<br />

Solution 2. We present a different proof of the estimate M N2N¡2 1. We do not use the<br />

induction, involving Hall’s lemma on matchings instead.<br />

Consider arbitrary 2 N¡2 1 distinct flags <strong>and</strong> arrange them into a largeÔ2N¡2 1Õ¢N<br />

flag. Construct two bipartite graphs G yÔVV½, E yÕ<strong>and</strong> G bÔVV½, E bÕwith the<br />

common set of vertices as follows. Let V <strong>and</strong> V½be the set of columns <strong>and</strong> the set of flags<br />

under consideration, respectively. Next, let the edgeÔc, fÕappear in E y if the intersection of<br />

column c <strong>and</strong> flag f is yellow, <strong>and</strong>Ôc, fÕÈE otherwise. Then we have to prove exactly that<br />

one of the graphs G y <strong>and</strong> G b contains a matching with all the vertices of V involved.<br />

b<br />

two sets of columns S y , S<br />

yÕ<br />

bV such thatE y<br />

yÔS yÕS flag is connected to c either in G y or in G b , hence E yÔS yÕE 2 N¡2 1V½E yÔS E bÔS bÕS S<br />

So, we have S yS b∅. Let yS bS yÔS yÕE<br />

Assume that these matchings do not exist. By Hall’s lemma, it means that there exist<br />

y¡1 <strong>and</strong>E bÔS bÕS b¡1 (in the<br />

left-h<strong>and</strong> sides, E yÔS yÕ<strong>and</strong> E bÔS bÕdenote respectively the sets of all vertices connected to S y<br />

<strong>and</strong> S b in the corresponding graphs). Our aim is to prove that this is impossible. Note that<br />

S y , S bV since N2N¡2 1.<br />

First, suppose that S yS b∅,<br />

V¾V½Þ<br />

so there exists some cÈS yS b . Note that each<br />

bÔS bÕV½. Hence we have<br />

b¡22N¡4; this is impossible for N4.<br />

y, b. From the construction of our graph,<br />

bÕ<br />

we have that all the flags in the set E bÔS bÕ¨have blue squares in the<br />

columns of S y <strong>and</strong> yellow squares in the columns of S b . Hence the only undetermined positions<br />

in these flags are the remaining N¡y¡b ones, so 2 N¡y¡bV¾V½¡E yÔS yÕ¡E bÔS 2 N¡2 1¡Ôy¡1Õ¡Ôb¡1Õ, or, denoting cy b, 2 N¡c c2N¡2 2. This is impossible<br />

since Nc2.


28<br />

C3. 2500 chess kings have to be placed on a 100¢100 chessboard so that<br />

(i) no king can capture any other one (i.e. no two kings are placed in two squares sharing<br />

a common vertex);<br />

(ii) each row <strong>and</strong> each column contains exactly 25 kings.<br />

Find the number of such arrangements. (Two arrangements differing by rotation or symmetry<br />

are supposed to be different.)<br />

Answer. There are two such arrangements.<br />

(Russia)<br />

Solution. Suppose that we have an arrangement satisfying the problem conditions. Divide the<br />

board into 2¢2 pieces; we call these pieces blocks. Each block can contain not more than one<br />

king (otherwise these two kings would attack each other); hence, by the pigeonhole principle<br />

each block must contain exactly one king.<br />

Now assign to each block a letter T or B if a king is placed in its top or bottom half,<br />

respectively. Similarly, assign to each block a letter L or R if a king st<strong>and</strong>s in its left or right<br />

half. So we define T-blocks, B-blocks, L-blocks, <strong>and</strong> R-blocks. We also combine the letters; we call<br />

a block a TL-block if it is simultaneously T-block <strong>and</strong> L-block. Similarly we define TR-blocks,<br />

BL-blocks, <strong>and</strong> BR-blocks. The arrangement of blocks determines uniquely the arrangement of<br />

kings; so in the rest of the solution we consider the 50¢50 system of blocks (see Fig. 1). We<br />

identify the blocks by their coordinate pairs; the pairÔi, jÕ, where 1i, j50, refers to the<br />

jth block in the ith row (or the ith block in the jth column). The upper-left block isÔ1, 1Õ.<br />

The system of blocks has the following properties..<br />

(i½) IfÔi, jÕis a B-block thenÔi 1, jÕis a B-block: otherwise the kings in these two blocks<br />

can take each other. Similarly: ifÔi, jÕis a T-block thenÔi¡1, jÕis a T-block; ifÔi, jÕis an<br />

L-block thenÔi, j¡1Õis an L-block; ifÔi, jÕis an R-block thenÔi, j 1Õis an R-block.<br />

(ii½) Each column contains exactly 25 L-blocks <strong>and</strong> 25 R-blocks, <strong>and</strong> each row contains<br />

exactly 25 T-blocks <strong>and</strong> 25 B-blocks. In particular, the total number of L-blocks (or R-blocks,<br />

or T-blocks, or B-blocks) is equal to 25¤501250.<br />

Consider any B-block of the formÔ1, jÕ. By (i½), all blocks in the jth column are B-blocks;<br />

so we call such a column B-column. By (ii½), we have 25 B-blocks in the first row, so we obtain<br />

25 B-columns. These 25 B-columns contain 1250 B-blocks, hence all blocks in the remaining<br />

columns are T-blocks,<br />

<br />

<strong>and</strong> we obtain 25 T-columns. Similarly, there are exactly 25 L-rows <strong>and</strong><br />

exactly 25 R-rows.<br />

Now consider an arbitrary pair of a T-column <strong>and</strong> a neighboring B-column (columns with<br />

numbers j <strong>and</strong> j 1).<br />

<br />

<br />

1 2 3<br />

j j+1<br />

1 TL BL TL<br />

i<br />

L<br />

2 TL BR TR<br />

i+1<br />

3 BL BL TR<br />

T B<br />

Fig. 1 Fig. 2<br />

Case 1. Suppose that the jth column is a T-column, <strong>and</strong> theÔj 1Õth column is a B-<br />

column. Consider some index i such that the ith row is an L-row; thenÔi, j 1Õis a BL-block.<br />

Therefore,Ôi 1, jÕcannot be a TR-block (see Fig. 2), henceÔi 1, jÕis a TL-block, thus the


29<br />

Ôi 1Õth row is an L-row. Now, choosing the ith row to be the topmost L-row, we successively<br />

obtain that all rows from the ith to the 50th are L-rows. Since we have exactly 25 L-rows, it<br />

follows that the rows from the 1st to the 25th are R-rows, <strong>and</strong> the rows from the 26th to the<br />

50th are L-rows.<br />

Now consider the neighboring R-row <strong>and</strong> L-row (that are the rows with numbers 25 <strong>and</strong><br />

26). Replacing in the previous reasoning rows by columns <strong>and</strong> vice versa, the columns from the<br />

1st to the 25th are T-columns, <strong>and</strong> the columns from the 26th to the 50th are B-columns. So<br />

we have a unique arrangement of blocks that leads to the arrangement of kings satisfying the<br />

condition of the problem (see Fig. 3).<br />

<br />

<br />

<br />

<br />

TR TR BR BR<br />

TR TR BR BR<br />

<br />

<br />

TL TL BL BL<br />

TL TL BL BL<br />

Fig. 3 Fig. 4<br />

Case 2. Suppose that the jth column is a B-column, <strong>and</strong> theÔj 1Õth column is a T-column.<br />

Repeating the arguments from Case 1, we obtain that the rows from the 1st to the 25th are<br />

L-rows (<strong>and</strong> all other rows are R-rows), the columns from the 1st to the 25th are B-columns<br />

(<strong>and</strong> all other columns are T-columns), so we find exactly one more arrangement of kings (see<br />

Fig. 4).


30<br />

C4. Six stacks S 1 , . . ., S 6 of coins are st<strong>and</strong>ing in a row. In the beginning every stack contains<br />

a single coin. There are two types of allowed moves:<br />

Move 1: If stack S k with 1k5contains at least one coin, you may remove one coin<br />

from S k <strong>and</strong> add two coins to S k 1.<br />

Move 2: If stack S k with 1k4contains at least one coin, then you may remove<br />

one coin from S k <strong>and</strong> exchange stacks S k 1 <strong>and</strong> S k 2.<br />

Decide whether it is possible to achieve by a sequence of such moves that the first five stacks<br />

are empty, whereas the sixth stack S 6 contains exactly 2010 20102010 coins.<br />

C4½. Same as Problem C4, but the constant 2010 20102010 is replaced by 2010 2010 .<br />

2, 0ÕÔa¡1,<br />

(Netherl<strong>and</strong>s)<br />

Answer. Yes (in both variants of the problem). There exists such a sequence of moves.<br />

Solution. Denote byÔa 1 , a 2 , . . .,a nÕÔa½1, a½2, . . .,a½nÕthe following: if some consecutive stacks<br />

contain a 1 , . . .,a n coins, then it is possible to perform several allowed moves such that the stacks<br />

contain a½1 , . . .,a½n coins respectively, whereas the contents of the other stacks remain unchanged.<br />

Let A2010 2010 or A2010 20102010 , respectively. Our goal is to show that<br />

Ô1, 1, 1, 1, 1, 1ÕÔ0, 0, 0, 0, 0, AÕ.<br />

First we prove two auxiliary observations.<br />

Lemma 1.Ôa, 0, 0ÕÔ0, 2 a , 0Õfor every a1.<br />

Proof. We prove by induction thatÔa, 0, 0ÕÔa¡k, 2 k , 0Õfor every 1ka. For k1,<br />

apply Move 1 to the first stack:<br />

Ôa, 0, 0ÕÔa¡1, 2 1 , 0Õ.<br />

Now assume that ka<strong>and</strong> the statement holds for some ka. Starting fromÔa¡k, 2 k , 0Õ,<br />

apply Move 1 to the middle stack 2 k times, until it becomes empty. Then apply Move 2 to the<br />

first stack:<br />

ÐÓÑÓÒ Ð<br />

Ôa¡k, 2 k , 0ÕÔa¡k, 2 k 1 , 0Õ.<br />

Hence,<br />

Ôa, 0, 0ÕÔa¡k, 2 k 1 , 0Õ. Lemma 2. For every positive integer n, let P n2 2...2<br />

(e.g. P 322216). Then<br />

n<br />

2 k¡1, 2Õ¤¤¤Ôa¡k,<br />

2 k , 0ÕÔa¡k¡1,<br />

0, 2 k 1ÕÔa¡k¡1,<br />

Ôa, 0, 0, 0ÕÔ0, P a , 0, 0Õfor every a1.<br />

Proof. Similarly to Lemma 1, we prove thatÔa, 0, 0, 0ÕÔa¡k,<br />

For k1, apply Move 1 to the first stack:<br />

Ôa, 0, 0, 0ÕÔa¡1,<br />

2, 0, 0ÕÔa¡1,<br />

P k , 0, 0Õfor every 1ka.<br />

P 1 , 0, 0Õ.<br />

Ð<br />

P k 1, 0, 0Õ.<br />

k 1, 0, 0Õ.<br />

Now assume that the lemma holds for some ka. Starting fromÔa¡k, P k , 0, 0Õ, apply<br />

Lemma 1, then apply Move 1 to the first stack:<br />

Therefore,<br />

Ôa¡k, P k , 0, 0ÕÔa¡k,<br />

0, 2 P k<br />

, 0ÕÔa¡k,<br />

Ôa, 0, 0, 0ÕÔa¡k,<br />

P k , 0, 0ÕÔa¡k¡1,<br />

0, P k 1, 0ÕÔa¡k¡1,<br />

P


31<br />

0Õ<br />

Now we prove the statement of the problem.<br />

First apply Move 1 to stack 5, then apply Move 2 to stacks S 4 , S 3 , S 2 <strong>and</strong> S 1 in this order.<br />

Then apply Lemma 2 twice:<br />

Ô1, 1, 1, 1, 1, 1ÕÔ1, 1, 1, 1, 0, 3ÕÔ1, 1, 1, 0, 3, 0ÕÔ1, 1, 0, 3, 0, 0ÕÔ1, 0, 3, 0, 0,<br />

Ô0, 3, 0, 0, 0, 0ÕÔ0, 0, P 3 , 0, 0, 0ÕÔ0, 0, 16, 0, 0, 0ÕÔ0, 0, 0, P 16 , 0, 0Õ.<br />

We already have more than A coins in stack S 4 , since<br />

A2010 20102010Ô2 11Õ201020102 11¤201020102 201020112Ô211Õ20112 211¤20112 2215P 16 .<br />

To decrease the number of coins in stack S 4<br />

0Õ<br />

, apply Move 2 to this stack repeatedly until its<br />

size decreases to Aß4. (In every step, we remove a coin from S 4 <strong>and</strong> exchange the empty stacks<br />

0, 0, P 16¡1, 0, 0ÕÔ0, 0, 0, P 16¡2, 0,<br />

¤¤¤Ô0, 0, 0, Aß4, 0, 0Õ.<br />

Finally, apply Move 1 repeatedly to empty stacks S 4 <strong>and</strong> S 5 :<br />

Ô0, 0, 0, Aß4, 0, 0Õ¤¤¤Ô0, 0, 0, 0, Aß2, 0Õ¤¤¤Ô0, 0, 0, 0, 0, AÕ.<br />

Comment 1. Starting with only 4 stack, it is not hard to check manually that we can achieve at<br />

most 28 coins in the last position. However, around 5 <strong>and</strong> 6 stacks the maximal number of coins<br />

Ô1,1,1,1,1,1ÕÔ0,3,1,1,1,1ÕÔ0,1,5,1,1,1ÕÔ0,1,1,9,1,1Õ<br />

explodes. With 5 stacks it is possible to achieve more than 2 214 coins. With 6 stacks the maximum is<br />

greater than P . P2 14<br />

It is not hard to show that the numbers 2010 2010 <strong>and</strong> 2010 20102010 in the problem can be replaced<br />

by any nonnegative integer up to P . P2 14<br />

Comment 2. The simpler variant C4½of the problem can be solved without Lemma 2:<br />

Ô0,1,1,1,17,1ÕÔ0,1,1,1,0,35ÕÔ0,1,1,0,35,0ÕÔ0,1,0,35,0,0Õ<br />

Ô0,0,35,0,0,0ÕÔ0,0,1,2 34 ,0,0ÕÔ0,0,1,0,2 234 ,0ÕÔ0,0,0,2 ,0,0Õ<br />

234<br />

S 5 <strong>and</strong> S 6 .)Ô0, 0, 0, P 16 , 0, 0ÕÔ0,<br />

Ô0,0,0,2 234¡1,0,0Õ...Ô0,0,0,Aß4,0,0ÕÔ0,0,0,0,Aß2,0ÕÔ0,0,0,0,0,AÕ.<br />

For this reason, the PSC suggests to consider the problem C4 as well. Problem C4 requires more<br />

invention <strong>and</strong> technical care. On the other h<strong>and</strong>, the problem statement in C4½hides the fact that the<br />

resulting amount of coins can be such incredibly huge <strong>and</strong> leaves this discovery to the students.


32<br />

C5. n4 players participated in a tennis tournament. Any two players have played exactly<br />

one game, <strong>and</strong> there was no tie game. We call a company of four players bad if one player<br />

was defeated by the other three players, <strong>and</strong> each of these three players won a game <strong>and</strong> lost<br />

another game among themselves. Suppose that there is no bad company in this tournament.<br />

Let w i <strong>and</strong> l i be respectively the number of wins <strong>and</strong> losses of the ith player. Prove that<br />

nôi1Ôw i¡l iÕ30. (1)<br />

(South Korea)<br />

Solution. For any tournament T satisfying the problem condition, denote by SÔTÕsum under<br />

consideration, namely<br />

SÔTÕnôi1Ôw i¡l iÕ3 .<br />

4<br />

4<br />

First, we show that the statement holds if a tournament T has only 4 players. Actually, let<br />

AÔa 1 , a 2 , a 3 , a 4Õbe the number of wins of the players; we may assume that a 1a 2a 3a 4 .<br />

We have a 1 a 2 a 3 a 2¨6, hence a 41. If a 40, then we cannot have<br />

a 1a 2a 32, otherwise the company of all players is bad. Hence we should have<br />

AÔ3, 2, 1, 0Õ, <strong>and</strong> SÔTÕ33 1 3 Ô¡1Õ3 Ô¡3Õ30. On the other h<strong>and</strong>, if a 41, then<br />

only two possibilities, AÔ3, 1, 1, 1Õ<strong>and</strong> AÔ2, 2, 1, 1Õcan take place. In the former case we<br />

have SÔTÕ33 3¤Ô¡2Õ30, while in the latter one SÔTÕ13 1 3 Ô¡1Õ3 SÔTÕô<br />

Ô¡1Õ30, as<br />

desired.<br />

Now we turn to the general problem. Consider a tournament T with no bad companies <strong>and</strong><br />

enumerate the players by the numbers from 1 to n. For every 4 players i 1 , i 2 , i 3 , i 4 consider a<br />

“sub-tournament” T i1 i 2 i 3 i 4<br />

consisting of only these players <strong>and</strong> the games which they performed<br />

with each other. By the abovementioned, we have SÔT i1 i 2 i 3 i 4Õ0. Our aim is to prove that<br />

SÔT i1 i 2 i 3 i 4Õ, (2)<br />

i 1 ,i 2 ,i 3 ,i 4<br />

ε ij«3<br />

ôε ij1 ε ij2 ε ij3 .<br />

j 1 ,j 2 ,j 3i<br />

where the sum is taken over all 4-tuples of distinct numbers from the setØ1, . . .,nÙ. This way<br />

the problem statement will be established.<br />

We interpret the numberÔw i¡l iÕ3 as following. For ij, let ε ij1 if the ith player wins<br />

against the jth one, <strong>and</strong> ε ij¡1 otherwise. Then<br />

Ôw i¡l iÕ3£ôji SÔTÕ ô<br />

Hence,<br />

ij1 ε ij2 ε ij3 .<br />

,j 2 ,j 3Ùε<br />

SÔTÕ ô<br />

iÊØj 1<br />

To simplify this expression, consider all the terms in this sum where two indices are equal.<br />

Øi,j 1 ,j 2 ,j 3Ù4<br />

If, for instance, j 1j 2 , then the term contains ε 2 ij 11, so we can replace this term by ε ij3 .<br />

Make such replacements for each such term; obviously, after this change each term of the form<br />

ε ij3 will appear PÔTÕtimes, hence<br />

PÔTÕS 2ÔTÕ.<br />

ε<br />

1ÔTÕ ij1 ε ij2 ε ij3 ε PÔTÕôij<br />

ijS


33<br />

We show that S 2ÔTÕ0<strong>and</strong> hence SÔTÕS 1ÔTÕfor each tournament. Actually, note that<br />

ε ij¡ε ji , <strong>and</strong> the whole sum can<br />

1ÔTÕô<br />

be split into such pairs. Since the sum in each pair is 0, so<br />

is S 2ÔTÕ.<br />

Thus the desired equality (2) rewrites as<br />

S S 1ÔT i1 i 2 i 3 i 4Õ. (3)<br />

i 1 ,i 2 ,i 3 ,i 4<br />

Now, if all the numbers j 1 , j 2 , j 3 are distinct, then the setØi, j 1 , j 2 , j 3Ùis contained in exactly<br />

one 4-tuple, hence the term ε ij1 ε ij2 ε ij3 appears in the right-h<strong>and</strong> part of (3) exactly once, as<br />

well as in the left-h<strong>and</strong> part. Clearly, there are no other terms in both parts, so the equality is<br />

established.<br />

Solution 2. Similarly to the first solution, we call the subsets of players as companies, <strong>and</strong><br />

the k-element subsets will be called as k-companies.<br />

In any company of the players, call a player the local champion of the company if he defeated<br />

all other members of the company. Similarly, if a player lost all his games against the others<br />

in the company then call him the local loser of the company. Obviously every company has<br />

at most one local champion <strong>and</strong> at most one local loser. By the condition of the problem,<br />

whenever a 4-company has a local loser, then this company has a local champion as well.<br />

Suppose that k is some positive integer, <strong>and</strong> let us count all cases when a player is the local<br />

champion of some k-company. The ith player won against w i other player. To be the local<br />

champion of a k-company, he must be a member of the company, <strong>and</strong> the other k¡1 members<br />

must be chosen from those whom he defeated. Therefore, the ith player is the local champion<br />

of¢w i<br />

k¡1Åk-companies. Hence, the total number of local champions of all k-companies is<br />

nôi1¢w i<br />

k¡1Å.<br />

Similarly, the total number of local losers of the k-companies is<br />

Now apply this for k2, 3 <strong>and</strong> 4.<br />

Since every game has a winner <strong>and</strong> a loser, we have<br />

w<br />

nôi1<br />

inôi1<br />

nôi1¢l i<br />

k¡1Å.<br />

i¢n<br />

l <strong>and</strong> hence<br />

2Å,<br />

nôi1 w i¡l i¨0. (4)<br />

In every 3-company, either the players defeated one another in a cycle or the company has<br />

both a local champion <strong>and</strong> a local loser. Therefore, the total number of local champions <strong>and</strong><br />

nôi1¢w i<br />

local losers in the 3-companies is the same,<br />

2Ånôi1¢l i<br />

2Å. So we have<br />

nôi1£¢w 2Å¡¢l i<br />

2Å«0. i<br />

(5)<br />

In every 4-company, by the problem’s condition, the number of local losers is less than or<br />

equal to the number of local champions. Then the same holds for the total numbers of local


34<br />

champions <strong>and</strong> local losers in all 4-companies, so<br />

nôi1¢w i<br />

3Ånôi1¢l i<br />

3Å. Hence,<br />

3Å«<br />

nôi1£¢w 3Å¡¢l i<br />

3Å«0. i<br />

(6)<br />

2Å«¡<br />

3Å«<br />

Ôx¡yÕ324£¢x 3Å¡¢y 24£¢x 2Å¡¢y<br />

2Å«¡<br />

3Ôx yÕ2¡4¨Ôx¡yÕ.<br />

iÕ324£¢w 3Å¡¢l<br />

Ôw i<br />

24£¢w 2Å¡¢l i i<br />

i¡l ÐÓÓÓÓÓÓÓÓÓÑÓÓÓÓÓÓÓÓÓÒ 3Å« ÐÓÓÓÓÓÓÓÓÓÑÓÓÓÓÓÓÓÓÓÒ 2Å«<br />

3Ôn¡1Õ2¡4¨¡w<br />

ÐÓÓÓÓÓÑÓÓÓÓÓÒ<br />

i¡l i©.<br />

nôi1£¢w nôi1£¢w 3Å¡¢l i i<br />

2Å¡¢l i i<br />

Now we establish the problem statement (1) as a linear combination of (4), (5) <strong>and</strong> (6). It<br />

is easy check that<br />

Apply this identity to xw 1 <strong>and</strong> yl i . Since every player played n¡1 games, we have<br />

w i l in¡1, <strong>and</strong> thus<br />

i<br />

i©<br />

Then<br />

nôi1Ôw i¡l iÕ324<br />

3Ôn¡1Õ2¡4¨nôi1¡w i¡l 0.<br />

0<br />

24<br />

0<br />

¡<br />

0


35<br />

C6. Given a positive integer k <strong>and</strong> other two integers bw1. There are two strings of<br />

pearls, a string of b black pearls <strong>and</strong> a string of w white pearls. The length of a string is the<br />

number of pearls on it.<br />

One cuts these strings in some steps by the following rules. In each step:<br />

(i) The strings are ordered by their lengths in a non-increasing order. If there are some<br />

strings of equal lengths, then the white ones precede the black ones. Then k first ones (if they<br />

consist of more than one pearl) are chosen; if there are less than k strings longer than 1, then<br />

one chooses all of them.<br />

(ii) Next, one cuts each chosen string into two parts differing in length by at most one.<br />

(For instance, if there are strings of 5, 4, 4, 2 black pearls, strings of 8, 4, 3 white pearls <strong>and</strong><br />

k4, then the strings of 8 white, 5 black, 4 white <strong>and</strong> 4 black pearls are cut into the parts<br />

Ô4, 4Õ,Ô3, 2Õ,Ô2, 2Õ<strong>and</strong>Ô2, 2Õ, respectively.)<br />

The process stops immediately after the step when a first isolated white pearl appears.<br />

Prove that at this stage, there will still exist a string of at least two black pearls.<br />

(Canada)<br />

Solution 1. Denote the situation after the ith step by A i ; hence A 0 is the initial situation, <strong>and</strong><br />

A i¡1A i is the ith step. We call a string containing m pearls an m-string; it is an m-w-string<br />

or a m-b-string if it is white or black, respectively.<br />

We continue the process until every string consists of a single pearl. We will focus on three<br />

moments of the process: (a) the first stage A s when the first 1-string (no matter black or<br />

white) appears; (b) the first stage A t where the total number of strings is greater than k (if<br />

such moment does not appear then we put t); <strong>and</strong> (c) the first stage A f when all black<br />

pearls are isolated. It is sufficient to prove that in A f¡1 (or earlier), a 1-w-string appears.<br />

We start with some easy properties of the situations under consideration. Obviously, we<br />

have sf. Moreover, all b-strings from A f¡1 become single pearls in the fth step, hence all<br />

of them are 1- or 2-b-strings.<br />

Next, observe that in each step A iA i 1 with it¡1, allÔ1Õ-strings were cut since<br />

there are not more than k strings at all; if, in addition, is, then there were no 1-string, so<br />

all the strings were cut in this step.<br />

Now, let B i <strong>and</strong> b i be the lengths of the longest <strong>and</strong> the shortest b-strings in A i , <strong>and</strong><br />

let W i <strong>and</strong> w i be the same for w-strings. We show by induction on iminØs, tÙthat (i) the<br />

situation A i contains exactly 2 i black <strong>and</strong> 2 i white strings, (ii) B iW i , <strong>and</strong> (iii) b iw i .<br />

The base case i0is obvious. For the induction step, if iminØs, tÙthen in the ith step,<br />

each string is cut, thus the claim (i) follows from the induction hypothesis; next, we have<br />

B iÖB i¡1ß2×ÖW i¡1ß2×W i <strong>and</strong> b iØb i¡1ß2ÙØw i¡1ß2Ùw i , thus establishing (ii)<br />

<strong>and</strong> (iii).<br />

For the numbers s, t, f, two cases are possible.<br />

Case 1. Suppose that stor ft 1 (<strong>and</strong> hence st 1); in particular, this is true<br />

when t. Then in A s¡1 we have B s¡1W s¡1, b s¡1w s¡11as s¡1minØs, tÙ.<br />

Now, if sf, then in A s¡1, there is no 1-w-string as well as noÔ2Õ-b-string. That is,<br />

2B s¡1W s¡1b s¡1w s¡11, hence all these numbers equal 2. This means that<br />

in A s¡1, all strings contain 2 pearls, <strong>and</strong> there are 2 s¡1 black <strong>and</strong> 2 s¡1 white strings, which<br />

means b2¤2s¡1w. This contradicts the problem conditions.<br />

Hence we have sf¡1 <strong>and</strong> thus st. Therefore, in the sth step each string is cut<br />

into two parts. Now, if a 1-b-string appears in this step, then from w s¡1b s¡1 we see that a


36<br />

1-w-string appears as well; so, in each case in the sth step a 1-w-string appears, while not all<br />

black pearls become single, as desired.<br />

Case 2. Now assume that t 1s<strong>and</strong> t 2f. Then in A t we have exactly 2 t white<br />

<strong>and</strong> 2 t black strings, all being larger than 1, <strong>and</strong> 2 t 1k2t (the latter holds since 2 t is the<br />

total number of strings in A t¡1). Now, in theÔt 1Õst step, exactly k strings are cut, not more<br />

than 2 t of them being black; so the number of w-strings in A t 1 is at least 2 t Ôk¡2tÕk.<br />

Since the number of w-strings does not decrease in our process, in A f¡1 we have at least k<br />

white strings as well.<br />

Finally, in A f¡1, all b-strings are not larger than 2, <strong>and</strong> at least one 2-b-string is cut in<br />

the fth step. Therefore, at most k¡1 white strings are cut in this step, hence there exists a<br />

w-string W which is not cut in the fth step. On the other h<strong>and</strong>, since a 2-b-string is cut, all<br />

Ô2Õ-w-strings should also be cut in the fth step; hence W should be a single pearl. This is<br />

exactly what we needed.<br />

Comment. In this solution, we used the condition bw only to avoid the case bw2 t . Hence,<br />

if a number bw is not a power of 2, then the problem statement is also valid.<br />

Solution 2. We use the same notations as introduced in the first paragraph of the previous<br />

solution. We claim that at every stage, there exist a u-b-string <strong>and</strong> a v-w-string such that<br />

either<br />

(i) uv1, or<br />

(ii) 2uv2u, <strong>and</strong> there also exist k¡1 ofÔvß2Õ-strings other than considered<br />

above.<br />

First, we notice that this statement implies the problem statement. Actually, in both<br />

cases (i) <strong>and</strong> (ii) we have u1, so at each stage there exists aÔ2Õ-b-string, <strong>and</strong> for the last<br />

stage it is exactly what we need.<br />

Now, we prove the claim by induction on the number of the stage. Obviously, for A 0 the<br />

condition (i) holds since bw. Further, we suppose that the statement holds for A i , <strong>and</strong> prove<br />

it for A i 1. Two cases are possible.<br />

Case 1. Assume that in A i , there are a u-b-string <strong>and</strong> a v-w-string with uv. We can<br />

assume that v is the length of the shortest w-string in A i ; since we are not at the final stage,<br />

we have v2. Now, in theÔi 1Õst step, two subcases may occur.<br />

Subcase 1a. Suppose that either no u-b-string is cut, or both some u-b-string <strong>and</strong> some<br />

v-w-string are cut. Then in A i 1, we have either a u-b-string <strong>and</strong> aÔvÕ-w-string (<strong>and</strong> (i) is<br />

valid), or we have aÖuß2×-b-string <strong>and</strong> aØvß2Ù-w-string. In the latter case, from uv we get<br />

Öuß2×Øvß2Ù, <strong>and</strong> (i) is valid again.<br />

Subcase 1b. Now, some u-b-string is cut, <strong>and</strong> no v-w-string is cut (<strong>and</strong> hence all the strings<br />

which are cut are longer than v). If u½Öuß2×v, then the condition (i) is satisfied since we<br />

have a u½-b-string <strong>and</strong> a v-w-string in A i 1. Otherwise, notice that the inequality uv2<br />

implies u½2. Furthermore, besides a fixed u-b-string, other k¡1 ofÔv 1Õ-strings should<br />

be cut in theÔi 1Õst step, hence providing at least k¡1 ofÔÖÔv 1Õß2×Õ-strings, <strong>and</strong><br />

ÖÔv 1Õß2×vß2. So, we can put v½v, <strong>and</strong> we have u½vu2u½, so the condition (ii)<br />

holds for A i 1.<br />

Case 2. Conversely, assume that in A i there exist a u-b-string, a v-w-string (2uv2u)<br />

<strong>and</strong> a set S of k¡1 other strings larger than vß2 (<strong>and</strong> hence larger than 1). In theÔi 1Õst<br />

step, three subcases may occur.<br />

Subcase 2a. Suppose that some u-b-string is not cut, <strong>and</strong> some v-w-string is cut. The latter<br />

one results in aØvß2Ù-w-string, we have v½Øvß2Ùu, <strong>and</strong> the condition (i) is valid.


37<br />

Subcase 2b. Next, suppose that no v-w-string is cut (<strong>and</strong> therefore no u-b-string is cut as<br />

uv). Then all k strings which are cut have the lengthv, so each one results in aÔvß2Õstring.<br />

Hence in A i 1, there exist kk¡1 ofÔvß2Õ-strings other than the considered u- <strong>and</strong><br />

v-strings, <strong>and</strong> the condition (ii) is satisfied.<br />

Subcase 2c. In the remaining case, all u-b-strings are cut. This means that allÔuÕ-strings<br />

are cut as well, hence our v-w-string is cut. Therefore in A i 1 there exists aÖuß2×-b-string<br />

together with aØvß2Ù-w-string. Now, if u½Öuß2×Øvß2Ùv½then the condition (i) is<br />

fulfilled. Otherwise, we have u½v½u2u½. In this case, we show that u½2. If, to the<br />

contrary, u½1 (<strong>and</strong> hence u2), then all black <strong>and</strong> whiteÔ2Õ-strings should be cut in the<br />

Ôi 1Õst step, <strong>and</strong> among these strings there are at least a u-b-string, a v-w-string, <strong>and</strong> k¡1<br />

strings in S (k 1 strings altogether). This is impossible.<br />

Hence, we get 2u½v½2u½. To reach (ii), it remains to check that in A i 1, there exists<br />

a set S½of k¡1 other strings larger than v½ß2. These will be exactly the strings obtained from<br />

the elements of S. Namely, each sÈS was either cut in theÔi 1Õst step, or not. In the former<br />

case, let us include into S½the largest of the strings obtained from s; otherwise we include s<br />

itself into S½. All k¡1 strings in S½are greater than vß2v½, as desired.


38<br />

C7. Let P 1 , . . .,P s be arithmetic progressions of integers, the following conditions being<br />

satisfied:<br />

(i) each integer belongs to at least one of them;<br />

(ii) each progression contains a number which does not belong to other progressions.<br />

Denote by n the least common multiple of steps of these progressions; let np α 1<br />

1 . . .p α k<br />

k<br />

be<br />

its prime factorization. Prove that<br />

s1 kôi1α iÔp i¡1Õ.<br />

N<br />

(Germany)<br />

Solution 1. First, we prove the key lemma, <strong>and</strong> then we show how to apply it to finish the<br />

k grid we mean the set<br />

ØÔa 1 , . . ., a kÕ:a iÈZ, 0a in i¡1Ù; the elements of N will be referred to as points. In this<br />

grid, we define a subgrid as a subset of the form<br />

: b i1x i1 , . . .,b itx itÙ, (1)<br />

where IØi 1 , . . .,i tÙis an arbitrary nonempty set of indices, <strong>and</strong> x ijÈÖ0, n ij¡1×(1jt)<br />

are fixed integer numbers. Further, we say that a subgrid (1) is orthogonal to the ith coordinate<br />

axis if iÈI, <strong>and</strong> that it is parallel to the ith coordinate axis otherwise.<br />

Lemma. Assume that the grid N is covered by subgrids L 1 , L 2 , . . .,L s (this means Näs<br />

i1 L i)<br />

so that<br />

(ii½) each subgrid contains a point which is not covered by other subgrids;<br />

(iii) for each coordinate axis, there exists a subgrid L i orthogonal to this axis.<br />

Then<br />

solution.<br />

Let n 1 , . . .,n k be positive integers. By an n 1¢n 2¢¤¤¤¢n<br />

LØÔb 1 , . . .,b kÕÈN<br />

s1 kôi1Ôn i¡1Õ.<br />

Proof. Assume to the contrary that siÔn i¡1Õs½. Our aim is to find a point that is not<br />

covered by L 1 , . . ., L s .<br />

The idea of the proof is the following. Imagine that we exp<strong>and</strong> each subgrid to some maximal<br />

subgrid so that for the ith axis, there will be at most n i¡1 maximal subgrids orthogonal to<br />

this axis. Then the desired point can be found easily: its ith coordinate should be that not<br />

covered by the maximal subgrids orthogonal to the ith axis. Surely, the conditions for existence<br />

of such expansion are provided by Hall’s lemma on matchings. So, we will follow this direction,<br />

although we will apply Hall’s lemma to some subgraph instead of the whole graph.<br />

Construct a bipartite graph GÔVV½, EÕas follows. Let VØL 1 , . . .,L sÙ, <strong>and</strong> let<br />

V½Øv ij : 1is, 1jn i¡1Ùbe some set of s½elements. Further, let the edgeÔL m , v<br />

appear iff L m is orthogonal to the ith axis.<br />

For each subset WV , denote<br />

ijÕ<br />

fÔWÕØvÈV½:ÔL, vÕÈE for some LÈWÙ.<br />

Notice that fÔVÕV½by (iii).<br />

Now, consider the set WV containing the maximal number of elements such thatW<br />

fÔWÕ; if there is no such set then we set W∅. Denote W½fÔWÕ, UVÞW, U½V½ÞW½.


39<br />

By our<br />

WXW<br />

assumption <strong>and</strong><br />

XfÔWÕ<br />

the Lemma condition,fÔVÕV½V, hence WV <strong>and</strong> U∅.<br />

Permuting the coordinates, we can assume that U½Øv ij : 1ilÙ, W½Øv ij : l 1ikÙ.<br />

Consider the induced subgraph G½of G on the vertices UU½. We claim that for every<br />

XU, we getfÔXÕU½X(so G½satisfies the conditions of Hall’s lemma). Actually, we<br />

haveWfÔWÕ, so ifXfÔXÕU½for some XU, then we have<br />

fÔXÕU½fÔWÕÔfÔXÕU½ÕfÔWXÕ.<br />

This contradicts the maximality ofW.<br />

Thus, applying Hall’s lemma, we can assign to each LÈU some vertex v ijÈU½so that to<br />

distinct elements of U, distinct vertices of U½are assigned. In this situation, we say that LÈU<br />

corresponds to the ith axis, <strong>and</strong> write gÔLÕi. Since there are n i¡1 vertices of the form v ij ,<br />

we get that for each 1il, not more than n i¡1 subgrids correspond to the ith axis.<br />

Finally, we are ready to present the desired point. Since WV , there exists a point<br />

bÔb 1 , b 2 , . . .,b kÕÈNÞÔLÈWLÕ. On the other h<strong>and</strong>, for every 1il, consider any subgrid<br />

LÈU with gÔLÕi. This means exactly that L is orthogonal to the ith axis, <strong>and</strong> hence all<br />

its elements have the same ith coordinate c L . Since there are at most n i¡1 such subgrids,<br />

there exists a number 0a in i¡1 which is not contained in a setØc L : gÔLÕiÙ. Choose<br />

such number for every 1il. Now we claim that point aÔa 1 , . . .,a l , b l 1, . . .,b kÕis not<br />

covered, hence contradicting the Lemma condition.<br />

Surely, point a cannot lie in some LÈU, since all the points in L have gÔLÕth coordinate<br />

c La gÔLÕ. Ð<br />

On the other h<strong>and</strong>, suppose that aÈL for some LÈW; recall that bÊL. But the<br />

points a <strong>and</strong> b differ only at first l coordinates, so L should be orthogonal to at least one of<br />

the first l axes, <strong>and</strong> hence our graph contains some edgeÔL, v ijÕfor il. It contradicts the<br />

definition of W½. The Lemma is proved.<br />

Now we turn to the problem. Let d j be the step of the progression P j . Note that since<br />

nl.c.m.Ôd ÐÓÓÓÓÓÓÑÓÓÓÓÓÓÒ<br />

1 , . . .,d sÕ, for each<br />

ÐÓÓÓÓÓÓÑÓÓÓÓÓÓÒ<br />

1ik there exists an index jÔiÕsuch that p α i<br />

i§d jÔiÕ. rÔmÕ<br />

We<br />

assume that n1; otherwise the problem statement is trivial.<br />

For each 0mn¡1 <strong>and</strong> 1ik, let m i be the residue of m modulo p α i<br />

i , <strong>and</strong> let<br />

m ir iαi . . .r i1 be the base p i representation of m i (possibly, with some leading zeroes). Now,<br />

we put into correspondence to m the sequence rÔmÕÔr 11 , . . .,r 1α1 , r 21 , . . .,r kαkÕ. Hence<br />

lies in a p 1¢¤¤¤¢p 1 ¢¤¤¤¢p k¢¤¤¤¢p k grid N.<br />

α 1 times<br />

α k times<br />

Surely, if rÔmÕrÔm½Õthen p α i<br />

i§m i¡m½i , which follows pα i<br />

i§m¡m½for all 1ik;<br />

consequently, n§m¡m½. So, when m runs over the setØ0, . . .,n¡1Ù, the sequences rÔmÕdo<br />

not repeat; sinceNn, this means that r is a bijection betweenØ0, . . ., n¡1Ù<strong>and</strong> N. Now<br />

we will show that for each 1is, the set L iØrÔmÕ:mÈP iÙis a subgrid, <strong>and</strong> that for<br />

each axis there exists<br />

<br />

a subgrid orthogonal to this axis. Obviously, these subgrids cover N, <strong>and</strong><br />

the condition (ii½) follows directly from (ii). Hence the Lemma provides exactly the estimate<br />

we need.<br />

Consider some 1js<strong>and</strong> let d jp rÔqÕ<br />

γ 1<br />

1 . . .pγ k<br />

k . Consider some qÈP j <strong>and</strong> let<br />

Ôr 11 , . . ., r kαkÕ. Then for an arbitrary q½, setting rÔq½ÕÔr½11, . . .,r½kα kÕwe have<br />

p γ i<br />

i§q¡q½for each 1ik r i,tr½i,t for all tγ i .<br />

Hence L jØÔr½11 , . . .,r½kα kÕÈN : r i,tr½i,t for all tγ iÙwhich means that L j is a subgrid<br />

containing rÔqÕ. Moreover, in L jÔiÕ, all the coordinates corresponding to p i are fixed, so it is<br />

orthogonal to all of their axes, as desired.<br />

q½ÈP j


40<br />

Comment 1. The estimate in the problem is sharp for every n. One of the possible examples is the<br />

following one. For each 1ik, 0jα i¡1, 1kp¡1, let<br />

P i,j,kkp j i<br />

p j 1<br />

i<br />

Z,<br />

<strong>and</strong> add the progression P 0nZ. One can easily check that this set satisfies all the problem conditions.<br />

There also exist other examples.<br />

On the other h<strong>and</strong>, the estimate can be adjusted in the following sense. For every 1ik, let<br />

0α i0 ,α i1 ,... ,α ihi be all the numbers of the form ord piÔd jÕin an increasing order (we delete the<br />

repeating occurences of a number, <strong>and</strong> add a number 0α i0 if it does not occur). Then, repeating<br />

the arguments from the solution one can obtain that<br />

s1 kôi1<br />

h i ôj1Ôp α j¡α j¡1¡1Õ.<br />

Note that p α¡1αÔp¡1Õ, <strong>and</strong> the equality is achieved only for α1. Hence, for reaching the<br />

minimal number of the progressions, one should have α i,jj for all i,j. In other words, for each<br />

1jα i , there should be an index t such that ord piÔd tÕj.<br />

Solution 2. We start with introducing some notation. For positive integer r, we denote<br />

Ör×Ø1, 2, . . .,rÙ. Next, we say that a set of progressions PØP 1 , . . .,P sÙcover Z if each<br />

integer belongs to some of them; we say that this covering is minimal if no proper subset of P<br />

covers Z. Obviously, each covering contains a minimal subcovering.<br />

Next, for a minimal coveringØP 1 , . . .,P sÙ<strong>and</strong> for every 1is, let d i be the step of<br />

progression P i , <strong>and</strong> h i be some number which is contained in P i but in none of the other<br />

progressions. We assume that n1, otherwise the problem is trivial. This implies d i1,<br />

otherwise the progression P i covers all the numbers, <strong>and</strong> n1.<br />

tÙÖk×<br />

We will prove a more general statement, namely the following<br />

Claim. Assume that the progressions P 1 , . . .,P s <strong>and</strong> number np α 1<br />

1 . . .pα k<br />

k1 are chosen as<br />

in the problem statement. Moreover, choose some nonempty set of indices IØi 1 , . . .,i<br />

<strong>and</strong> some positive integer β iα i for every iÈI. Consider the set of indices<br />

Tj : 1js, <strong>and</strong> p α i¡β i 1<br />

i<br />

§d j for some iÈIµ.<br />

Then<br />

ôiÈI<br />

T1 β iÔp i¡1Õ. (2)<br />

Observe that the Claim for IÖk×<strong>and</strong> β iα i implies the problem statement, since the<br />

left-h<strong>and</strong> side in (2) is not greater than s. Hence, it suffices to prove the Claim.<br />

1. First, we prove the Claim assuming that all d j ’s are prime numbers. If for some 1ik<br />

we have at least p i progressions with the step p i , then they do not intersect <strong>and</strong> hence cover all<br />

the integers; it means that there are no other progressions, <strong>and</strong> np i ; the Claim is trivial in<br />

this case.<br />

Now assume that for every 1ik, there are not more than p i¡1 progressions with<br />

step p i ; each such progression covers the numbers with a fixed residue modulo p i , therefore<br />

there exists a residue q i mod p i which is not touched by these progressions. By the Chinese<br />

Remainder Theorem, there exists a number q such that qq iÔmod p iÕfor all 1ik; this<br />

number cannot be covered by any progression with step p i , hence it is not covered at all. A<br />

contradiction.


41<br />

2. Now, we assume that the general Claim is not valid, <strong>and</strong> hence we consider a counterexampleØP<br />

1 , . . .,P sÙfor the Claim; we can choose it to be minimal in the following sense:<br />

­the number n is minimal possible among all the counterexamples;<br />

­the sumi d i is minimal possible among all the counterexamples having the chosen value<br />

of n.<br />

As was mentioned above, not all numbers d i are primes; hence we can assume that d 1 is<br />

composite, say p 1§d 1 <strong>and</strong> d½1d 1<br />

p 11. Consider a progression P½1 having the step d½1, <strong>and</strong><br />

containing P 1 . We will focus on two coverings constructed as follows.<br />

(i) Surely, the progressions P½1, P 2 , . . .,P s cover Z, though this covering in not necessarily<br />

minimal. So, choose some minimal subcovering P½in it; surely P½1ÈP½since h 1 is not covered<br />

by P 2 , . . .,P s , so we may assume that P½ØP½1 P 2, . . ., P s½Ùfor some s½s. Furthermore, the<br />

,<br />

period of the covering P½can appear to be less than n; so we denote this period by<br />

l.c.m. d½1, d 2 , . . ., d s½¨.<br />

n½p α 1¡σ 1<br />

1 . . .p α k¡σ k<br />

k<br />

Observe that for each P jÊP½, we have h jÈP½1, otherwise h j would not be covered by P.<br />

(ii) On the other h<strong>and</strong>, each nonempty set of the form R iP iP½1 (1is) is also a<br />

progression with a step r il.c.m.Ôd i , d½1Õ, <strong>and</strong> such sets cover P½1. Scaling these progressions<br />

with the ratio 1ßd½1 , we obtain the progressions Q i with steps q ir ißd½1 which cover Z. Now we<br />

choose a minimal subcovering Q of this covering; again we should have Q 1ÈQ by the reasons<br />

of h 1 . Now, denote the period of Q by<br />

n¾l.c.m.Øq i : Q iÈQÙl.c.m.Ør i : Q iÈQÙ<br />

d½1<br />

p γ 1<br />

1 . . .p γ k<br />

k<br />

.<br />

d½1<br />

Note that if h jÈP½1 , then the image of h j under the scaling can be covered by Q j only; so, in<br />

this case we have Q jÈQ.<br />

Our aim is to find the desired number of progressions in coverings P <strong>and</strong> Q. First, we have<br />

nn½, <strong>and</strong> the sum of the steps in P½is less than that in P; hence the Claim is valid for P½.<br />

We apply it to the set of indices I½ØiÈI : β iσ iÙ<strong>and</strong> the exponents β½iβ i¡σ i ; hence<br />

the set under consideration is<br />

T½j : 1js½, <strong>and</strong> i¡σ pÔα iÕ¡β½i<br />

1p α i¡β i 1<br />

i i<br />

§d j for some iÈI½µTÖs½×,<br />

<strong>and</strong> we obtain that<br />

ôiÈIÔβ whereÔxÕ<br />

iÕ<br />

xÔx¡yÕ i¡1Õ1 i¡σ Ôp i¡1Õ,<br />

maxØx, 0Ù; the latter equality holds as for<br />

i¡1Õ<br />

iÊI½we have β iσ i .<br />

TTÖs½× TØs½<br />

Observe that<br />

TØs½<br />

minØx, yÙfor all x, y. So, if we find at least<br />

minØβ GôiÈI i , σ iÙÔp iÕ<br />

indices in 1, . . ., sÙ, then we would have<br />

1, . . .,<br />

ôiÈI sÙ1 Ôβ i¡σ minØβ i , σ<br />

ôiÈI<br />

iÙ¨Ôp i¡1Õ1 β iÔp i¡1Õ,<br />

ôiÈI½Ôβ TÖs½×T½1 i¡σ iÕÔp<br />

thus leading to a contradiction with the choice of P. We will find those indices among the<br />

indices of progressions in Q.


42<br />

3. Now denote I¾ØiÈI : σ i0Ù<strong>and</strong> consider some iÈI¾; then p α i<br />

the<br />

other h<strong>and</strong>, there exists an index jÔiÕsuch that p α i<br />

i§d jÔiÕ; this means that d jÔiÕЧn½<strong>and</strong> hence<br />

P jÔiÕcannot appear in P½, so jÔiÕs½. Moreover, we have observed before that in this case<br />

h jÔiÕÈP½1 , hence Q jÔiÕÈQ. This means that q jÔiÕ§n¾, therefore γ iα i for each iÈI¾(recall<br />

here that q ir ißd½1 <strong>and</strong> hence d jÔiÕ§r jÔiÕ§d½1n¾).<br />

Let d½1p τ 1<br />

1 . . .p τ k<br />

k<br />

. Then n¾p γ 1¡τ 1<br />

1 . . .p γ i¡τ i<br />

k<br />

. Now, if iÈI¾, then for every β the condition<br />

i¡τ pÔγ iÕ¡β 1<br />

i<br />

§q j is equivalent to p α i¡β 1<br />

i<br />

§r j .<br />

Note that n¾nßd½1n, hence we can apply the Claim to the covering Q. We perform<br />

this with the set of indices I¾<strong>and</strong> the exponents β¾iminØβ i , σ iÙ0. So, the set under<br />

consideration is<br />

iÙ iÙ iÈI¾µ<br />

T¾j : Q jÈQ, <strong>and</strong> i¡τ iÕ¡minØβ i<br />

T¾T<br />

,σ 1<br />

pÔγ i<br />

§q j<br />

Ø1ÙØs½<br />

for some<br />

j : Q jÈQ, <strong>and</strong> p α i¡minØβ i ,σ 1<br />

i<br />

§r j for some iÈI¾µ,<br />

<strong>and</strong> we obtainT¾1 G. Finally, we claim that 1, . . .,sÙ¨; then we<br />

will obtainTØs½<br />

iÙ iÙ<br />

1, . . .,sÙG, which is exactly what we need.<br />

To prove this, consider any jÈT¾. Observe first that α i¡minØβ i , σ 1α i¡σ iτ i ,<br />

hence from p α i¡minØβ i ,σ 1<br />

1<br />

i<br />

§d j , which means that<br />

jÈT. Next, the exponent of p i in d j is greater than that in n½, which means that P jÊP½. This<br />

may appear only if j1 or js½, as desired. This completes the proof.<br />

i<br />

§r jl.c.m.Ôd½1, d jÕwe have p α i¡minØβ i ,σ iÙ<br />

Comment 2. A grid analogue of the Claim is also valid. It reads as following.<br />

Claim. Assume that the grid N is covered by subgrids L 1 ,L 2 ,... ,L s so that<br />

(ii½) each subgrid contains a point which is not covered by other subgrids;<br />

(iii) for each coordinate axis, there exists a subgrid L i orthogonal to this axis.<br />

Choose some set of indices IØi 1 ,... ,i tÙÖk×, <strong>and</strong> consider the set of indices<br />

TØj : 1js,<strong>and</strong> L j is orthogonal to the ith axis for some iÈIÙ.<br />

Then ôiÈIÔn T1 i¡1Õ.<br />

This Claim may be proved almost in the same way as in Solution 1.<br />

iЧn½. On


Geometry<br />

G1. Let ABC be an acute triangle with D, E, F the feet of the altitudes lying on BC, CA, AB<br />

respectively. One of the intersection points of the line EF <strong>and</strong> the circumcircle is P. The<br />

lines BP <strong>and</strong> DF meet at point Q. Prove that APAQ.<br />

(United Kingdom)<br />

Solution 1. The line EF intersects the circumcircle at two points. Depending on the choice<br />

of P, there are two different cases to consider.<br />

Case 1: The point P lies on the ray EF (see Fig. 1).<br />

LetCABα,ABCβ <strong>and</strong>BCAγ. The quadrilaterals BCEF <strong>and</strong> CAFD are<br />

cyclic due to the right angles at D, E <strong>and</strong> F. So,<br />

BDF180¥¡FDCCAFα,<br />

AFE180¥¡EFBBCEγ,<br />

DFB180¥¡AFDDCAγ.<br />

Since P lies on the arc AB of the circumcircle,PBABCAγ. Hence, we have<br />

PBD BDFPBA ABD BDFγ β α180¥,<br />

<strong>and</strong> the point Q must lie on the extensions of BP <strong>and</strong> DF beyond the points P <strong>and</strong> F,<br />

respectively.<br />

From the cyclic quadrilateral APBC we get<br />

QPA180¥¡APBBCAγDFBQFA.<br />

Hence, the quadrilateral AQPF is cyclic. ThenAQP180¥¡PFAAFEγ.<br />

We obtained thatAQPQPAγ, so the triangle AQP is isosceles, APAQ.<br />

Q<br />

γ<br />

A<br />

A<br />

α<br />

P<br />

γ<br />

γ<br />

F γ<br />

γ<br />

E<br />

F<br />

γ<br />

γ<br />

γ<br />

E<br />

γ<br />

P<br />

B<br />

γ<br />

β<br />

α<br />

D<br />

γ<br />

C<br />

B<br />

Q<br />

D<br />

γ<br />

C<br />

Fig. 1 Fig. 2


45<br />

Case 2: The point P lies on the ray FE (see Fig. 2). In this case the point Q lies inside<br />

the segment FD.<br />

Similarly to the first case, we have<br />

QPABCAγDFB180¥¡AFQ.<br />

Hence, the quadrilateral AFQP is cyclic.<br />

ThenAQPAFPAFEγQPA. The triangle AQP is isosceles again,<br />

AQPQPA <strong>and</strong> thus APAQ.<br />

Comment. Using signed angles, the two possible configurations can be h<strong>and</strong>led simultaneously, without<br />

investigating the possible locations of P <strong>and</strong> Q.<br />

Solution 2. For arbitrary points X, Y on the circumcircle, denote byXY the central angle<br />

of the arc XY .<br />

Let P <strong>and</strong> P½be the two points where the line EF meets the circumcircle; let P lie on<br />

the arc AB <strong>and</strong> let P½lie on the arc CA. Let BP <strong>and</strong> BP½meet the line DF <strong>and</strong> Q <strong>and</strong> Q½,<br />

respectively (see Fig. 3). We will prove that APAP½AQAQ½.<br />

Q<br />

A<br />

P<br />

F<br />

γ<br />

γ<br />

γ<br />

E<br />

P ′<br />

Q ′<br />

γ<br />

B D<br />

C<br />

Fig. 3<br />

Like in the first solution, we haveAFEBFPDFBBCAγ from the<br />

cyclic quadrilaterals BCEF <strong>and</strong> CAFD.<br />

ByPB P½A2AFP½2γ2BCAAP PB, we have<br />

APP½A,PBAABP½<br />

Ô1Õ<br />

<strong>and</strong> APAP½.<br />

Ô2Õ<br />

Due toAPP½A, the lines BP <strong>and</strong> BQ½are symmetrical about line AB.<br />

Similarly, byBFPQ½FB, the lines FP <strong>and</strong> FQ½are symmetrical about AB. It<br />

follows that also the points P <strong>and</strong> P½are symmetrical to Q½<strong>and</strong> Q, respectively. Therefore,<br />

APAQ½<strong>and</strong> AP½AQ.<br />

The relations (1) <strong>and</strong> (2) together prove APAP½AQAQ½.


46<br />

G2. Point P lies inside triangle ABC. Lines AP, BP, CP meet the circumcircle of ABC<br />

again at points K, L, M, respectively. The tangent to the circumcircle at C meets line AB<br />

at S. Prove that SCSP if <strong>and</strong> only if MKML.<br />

(Pol<strong>and</strong>)<br />

Solution 1. We assume that CACB, so point S lies on the ray AB.<br />

From the similar triangles △PKM△PCA <strong>and</strong> △PLM△PCB we get<br />

KMPA PM<br />

CA<br />

<strong>and</strong><br />

PMCB LM . Multiplying these two equalities, we get<br />

PB<br />

LM<br />

KMCB<br />

CA¤PA<br />

PB .<br />

Hence, the relation MKML is equivalent to CB<br />

CAPB<br />

PA .<br />

Denote by E the foot of the bisector of angle B in triangle ABC. Recall that the locus of<br />

points X for which<br />

XBCA XA is the Apollonius circle Ω with the center Q on the line AB,<br />

CB<br />

<strong>and</strong> this circle passes through C <strong>and</strong> E. Hence, we have MKML if <strong>and</strong> only if P lies on Ω,<br />

that is QPQC.<br />

L<br />

C<br />

Ω<br />

K<br />

P<br />

A<br />

E<br />

M<br />

Fig. 1<br />

Now we prove that SQ, thus establishing the problem statement. We haveCES<br />

CAE ACEBCS ECBECS, so SCSE. Hence, the point S lies on AB<br />

as well as on the perpendicular bisector of CE <strong>and</strong> therefore coincides with Q.<br />

Solution 2. As in the previous solution, we assume that S lies on the ray AB.<br />

1. Let P be an arbitrary point inside both the circumcircle ω of the triangle ABC <strong>and</strong> the<br />

angle ASC, the points K, L, M defined as in the problem. We claim that SPSC implies<br />

MKML.<br />

Let E <strong>and</strong> F be the points of intersection of the line SP with ω, point E lying on the<br />

segment SP (see Fig. 2).<br />

B<br />

S


47<br />

F<br />

L<br />

ω<br />

P<br />

K<br />

C<br />

M<br />

E<br />

A<br />

B<br />

Fig. 2<br />

S<br />

We have SP 2SC2SA¤SB, so<br />

SBSA SP<br />

SP , <strong>and</strong> hence △PSA△BSP. Then<br />

BPSSAP. Since 2BPSBE LF <strong>and</strong> 2SAPBE EK we have<br />

LFEK. (1)<br />

On the other h<strong>and</strong>, fromSPCSCP we haveEC MFEC EM, or<br />

MFEM. (2)<br />

From (1) <strong>and</strong> (2) we getǑMFLMF FLME EKǑMEK <strong>and</strong> hence MKML.<br />

The claim is proved.<br />

2. We are left to prove the converse. So, assume that MKML, <strong>and</strong> introduce the<br />

points E <strong>and</strong> F as above. We have SC 2SE¤SF; hence, there exists a point P½lying on the<br />

segment EF such that SP½SC (see Fig. 3).<br />

L ′<br />

L<br />

C<br />

F<br />

ω<br />

P ′<br />

P<br />

K ′<br />

K<br />

E<br />

A<br />

B<br />

S<br />

M ′<br />

M<br />

Fig. 3


M½<br />

48<br />

Assume that PP½. Let the lines AP½, BP½, CP½meet ω again at points K½, L½,<br />

respectively. Now, if P½lies on the segment PF then by the first part of the solution we have<br />

ǑM½FL½ǑM½EK½. On the other h<strong>and</strong>, we haveǑMFLǑM½FL½ǑM½EK½ǑMEK, therefore<br />

ǑMFLǑMEK which contradicts MKML.<br />

Similarly, if point P½lies on the segment EP then we getǑMFLǑMEK which is impossible.<br />

Therefore, the points P <strong>and</strong> P½coincide <strong>and</strong> hence SPSP½SC.<br />

Solution 3. We present a different proof of the converse direction, that is, MKML<br />

SPSC. As in the previous solutions we assume that CACB, <strong>and</strong> line meets ω<br />

at E <strong>and</strong> F.<br />

From MLMK we getǑMEKǑMFL. Now we claim thatMEMF <strong>and</strong>EKFL.<br />

To the contrary, suppose first thatMEMF; thenEKǑMEK¡MEǑMFL¡MF<br />

FL. Now, the inequalityMEMF implies 2SCMEC MEEC MF2SPC<br />

<strong>and</strong> hence SPSC. On the other h<strong>and</strong>, the inequalityEKFL implies 2SPK<br />

EK AFFL AF2ABL, hence<br />

SPA180¥¡SPK180¥¡ABLSBP.<br />

L<br />

C<br />

F<br />

K<br />

P<br />

E<br />

A<br />

A ′<br />

B<br />

S<br />

M<br />

ω<br />

Fig. 4<br />

Consider the point A½on the ray SA for whichSPA½SBP; in our case, this point lies<br />

on the segment SA (see Fig. 4). Then △SBP△SPA½<strong>and</strong> SP 2SB¤SA½SB¤SASC 2 .<br />

Therefore, SPSC which contradicts SPSC.<br />

Similarly, one can prove that the inequalityMEMF is also impossible. So, we get<br />

MEMF <strong>and</strong> therefore 2SCMEC MEEC MF2SPC, which implies<br />

SCSP.


50<br />

G3. Let A 1 A 2 . . .A n be a convex polygon. Point P inside this polygon is chosen so that its<br />

projections P 1 , ..., P n onto lines A 1 A 2 , ..., A n A 1 respectively lie on the sides of the polygon.<br />

Prove that for arbitrary points X 1 , ..., X n on sides A 1 A 2 , ..., A n A 1 respectively,<br />

maxX 1 X 2<br />

P 1 P 2<br />

, . . ., X nX 1<br />

P n P 11.<br />

(Armenia)<br />

Solution 1. Denote P n 1P 1 , X n 1X 1 , A n 1A 1 .<br />

Lemma. Let point Q lies inside A 1 A 2 . . .A n . Then it is contained in at least one of the circumcircles<br />

of triangles X 1 A 2 X 2 , ..., X n A 1 X 1 .<br />

Proof. If Q lies in one<br />

2Õ ¤¤¤<br />

of the triangles X 1 A 2 X 2 , ..., X n A 1 X 1 , the claim is obvious. Otherwise<br />

Q lies inside the polygon X 1 X 2 . . .X n<br />

1Õ 1Õ ¤¤¤<br />

(see Fig. 1). Then we have<br />

X 1 QX ÔX n A 1 X 1 X n QX<br />

X n A 1 X ÔX 1 QX 2<br />

Ð<br />

2πnπ,<br />

hence there exists an index i such thatX i A i 1X i X 1 i QX i 1πn<br />

nπ. Since the<br />

quadrilateral QX i A i 1X i 1 is convex, this means exactly that Q is contained the circumcircle<br />

of △X i A i 1X i 1, as desired.<br />

ÔX 1 A 2 X 2<br />

¤¤¤ ÔX 1 A 1 X 2<br />

X n QX 1ÕÔn¡2Õπ<br />

Now we turn to the solution. Applying lemma, we get that P lies inside the circumcircle of<br />

triangle X i A i 1X i 1 for some i. Consider the circumcircles ω <strong>and</strong> Ω of triangles P i A i 1P i 1 <strong>and</strong><br />

X i A i 1X i 1 respectively (see Fig. 2); let r <strong>and</strong> R be their radii. Then we get 2rA i 1P2R<br />

(since P lies inside Ω), hence<br />

QED.<br />

P i P i 12r sinP i A i 1P i 12R sinX i A i 1X i 1X i X i 1,<br />

X 4<br />

A 4<br />

X 3<br />

Ω<br />

A 3<br />

P<br />

A 5 X i+1<br />

Q<br />

X 2 ω<br />

P i+1<br />

X 5<br />

A 1 X 1 A P i 2<br />

X i A i+1<br />

Fig. 1 Fig. 2


51<br />

Solution 2. As in Solution 1, we assume that all indices of points are considered modulo n.<br />

We will prove a bit stronger inequality, namely<br />

1 X 2<br />

maxX<br />

cosα 1 , . . ., X nX 1<br />

cosα n1,<br />

P 1 P 2 P n P 1<br />

where α i (1in) is the angle between lines X i X i 1 <strong>and</strong> P i P i 1. We denote β iA i P i P i¡1<br />

<strong>and</strong> γ iA i 1P i P i 1 for all 1in.<br />

i<br />

Suppose that for some 1in, point X i lies on the segment A i P i , while point X i 1 lies on<br />

the segment P i 1A i 2. Then the projection of the segment X i X i 1 onto the line P i P i 1 contains<br />

segment P i P i 1, since γ i <strong>and</strong> β i 1 are acute angles (see Fig. 3). Therefore, X i X i 1 cosα<br />

P i P i 1, <strong>and</strong> in this case the statement is proved.<br />

So, the only case left is when point X i lies on segment P i A i 1 for all 1in(the case<br />

when each X i lies on segment A i P i<br />

Y½ Y½<br />

1Y½<br />

is completely analogous).<br />

Now, assume to the contrary that the inequality<br />

X i X i 1 cosα iP i P i 1 (1)<br />

holds for every 1in. Let Y i <strong>and</strong> i 1 be the projections of X i <strong>and</strong> X i 1 onto P i P i 1. Then<br />

inequality (1) means exactly that Y i i 1P i P i 1, or P i Y iP i i 1 (again since γ i <strong>and</strong> β i 1<br />

are acute; see Fig. 4). Hence, we have<br />

X i P i cosγ iX i 1P i<br />

1¨<br />

1 cosβ i 1, 1in.<br />

Multiplying these inequalities, we get<br />

cos γ 1 cos γ 2¤¤¤cosγ ncosβ 1 cosβ 2¤¤¤cosβ n . (2)<br />

On the other h<strong>and</strong>, the sines theorem applied to triangle PP i P i 1 provides<br />

π<br />

PP i 2¡β i i 1<br />

.<br />

PP<br />

π<br />

i 1sin<br />

sin 2¡γ i¨cosβ γ i<br />

Multiplying these equalities we get<br />

which contradicts (2).<br />

1cos β 2<br />

cos γ 1¤cos β 3<br />

cos γ 2¤¤¤cosβ 1<br />

cosγ n<br />

α i<br />

X i+1<br />

P<br />

P i+1<br />

β i+1<br />

γ i<br />

X i P i A i+1<br />

P i+1<br />

γ i<br />

β i+1<br />

α i<br />

A i+1<br />

X i<br />

P<br />

X i−1<br />

X i+1<br />

Fig. 3 Fig. 4<br />

Y i<br />

Y ′<br />

i+1


52<br />

G4. Let I be the incenter of a triangle ABC <strong>and</strong> Γ be its circumcircle. Let the line AI<br />

intersect Γ at a point DA. Let F <strong>and</strong> E be points on side BC <strong>and</strong> arc BDC respectively<br />

such thatBAFCAE1<br />

2BAC. Finally, let G be the midpoint of the segment IF.<br />

Prove that the lines DG <strong>and</strong> EI intersect on Γ.<br />

(Hong Kong)<br />

Solution 1. Let X be the second point of intersection of line EI with Γ, <strong>and</strong> L be the foot<br />

of the bisector of angle BAC. Let G½<strong>and</strong> T be the points of intersection of segment DX with<br />

lines IF <strong>and</strong> AF, respectively. We are to prove that GG½, or IG½G½F. By the Menelaus<br />

theorem applied to triangle AIF <strong>and</strong> line DX, it means that we need the relation<br />

1G½F<br />

IG½TF<br />

AT¤AD<br />

ID , or TF<br />

ATID<br />

AD .<br />

Let the line AF intersect Γ at point KA(see Fig. 1); Furthermore,DCLDCB<br />

sinceBAKCAE we have<br />

BKCE, hence KE ‖ BC. Notice thatIATDAKEADEXDIXT, so<br />

the points I, A, X, T are concyclic. Hence we haveITAIXAEXAEKA, so<br />

IT ‖ KE ‖ BC. Therefore we obtain TF<br />

ATIL<br />

AI .<br />

Since CI is the bisector ofACL, we get<br />

AICL IL<br />

AC .<br />

DABCAD1<br />

2BAC, hence the triangles DCL <strong>and</strong> DAC are similar; therefore we get<br />

CL<br />

. Finally, it is known that the midpoint D of arc BC is equidistant from points I,<br />

ACDC<br />

AD<br />

B, C, hence DC<br />

ADID<br />

AD .<br />

Summarizing all these equalities, we get<br />

as desired.<br />

TF<br />

ATIL<br />

AICL<br />

ACDC<br />

ADID<br />

AD ,<br />

X<br />

A<br />

A<br />

I<br />

T<br />

I<br />

B<br />

C<br />

B<br />

F<br />

G ′<br />

L<br />

C<br />

D<br />

K<br />

E<br />

D<br />

L<br />

Fig. 1 Fig. 2


53<br />

Comment. The equality AI is known <strong>and</strong> can be obtained in many different ways. For<br />

ILAD<br />

DI<br />

instance, one can consider the inversion with center D <strong>and</strong> radius DCDI. This inversion takes<br />

<br />

ǑBAC to the segment BC, so point A goes to L. Hence<br />

DIAI IL<br />

<br />

, which is the desired equality.<br />

AD<br />

Solution 2. As in the previous solution, we introduce the points X, T <strong>and</strong> K <strong>and</strong> note that<br />

it suffice to prove the equality<br />

TF<br />

TF AT<br />

ATDI DI AD AT AF<br />

AD AT AD ADDI AD .<br />

SinceFADEAI <strong>and</strong>TDAXDAXEAIEA, we get that the triangles<br />

ATD <strong>and</strong> AIE are similar, therefore<br />

ADAI AT<br />

AE .<br />

Next, we also use the relation DBDCDI. Let J be the point on the extension<br />

of segment AD over point D such that DJDIDC (see Fig. 2). ThenDJC<br />

JCD1<br />

2Ôπ¡JDCÕ1<br />

2ADC1<br />

2ABCABI. Moreover,BAIJAC, hence<br />

triangles ABI <strong>and</strong> AJC are similar, so AB<br />

AJAI<br />

AC , or AB¤ACAJ¤AIÔDI ADÕ¤AI.<br />

<br />

On the other h<strong>and</strong>, we getABFABCAEC <strong>and</strong>BAFCAE, so triangles<br />

ABF <strong>and</strong> AEC are also similar, which implies AF , or AB¤ACAF¤AE.<br />

ACAB<br />

AE<br />

Summarizing we get<br />

AI AF<br />

AE <br />

AT AF<br />

AD<br />

ÔDI ADÕ¤AIAB¤ACAF¤AE<br />

as desired.<br />

Comment. In fact, point J is an excenter of triangle ABC.<br />

AD DI<br />

AD DI ,


54<br />

G5. Let ABCDE be a convex pentagon such that BC ‖ AE, ABBC AE, <strong>and</strong>ABC<br />

CDE. Let M be the midpoint of CE, <strong>and</strong> let O be the circumcenter of triangle BCD. Given<br />

thatDMO90¥, prove that 2BDACDE.<br />

(Ukraine)<br />

Solution 1. Choose point T on ray AE such that ATAB; then from AE ‖ BC we have<br />

CBTATBABT, so BT is the bisector ofABC. On the other h<strong>and</strong>, we have<br />

ETAT¡AEAB¡AEBC, hence quadrilateral<br />

BDCCDEABC180¥¡<br />

BCTE is a parallelogram, <strong>and</strong> the<br />

midpoint M of its diagonal CE is also the midpoint of the other diagonal BT.<br />

Next, let point K be symmetrical to D with respect to M. Then OM is the perpendicular<br />

bisector of segment DK, <strong>and</strong> hence ODOK, which means that point K lies on the circumcircle<br />

of triangle BCD. Hence we haveBDCBKC. On the other h<strong>and</strong>, the angles<br />

BKC <strong>and</strong> TDE are symmetrical with respect to M, soTDEBKCBDC.<br />

Therefore,BDTBDE EDTBDE<br />

BAT. This means that the points A, B, D, T are concyclic, <strong>and</strong> henceADBATB<br />

1<br />

2ABC1<br />

2CDE, as desired.<br />

B<br />

C<br />

B<br />

C<br />

O 2ϕ α<br />

D<br />

β<br />

α + β<br />

K<br />

M<br />

2ϕ − β − γ<br />

D<br />

γ<br />

A E<br />

T<br />

A<br />

E<br />

Solution 2. LetCBDα,BDCβ,ADEγ, <strong>and</strong>ABCCDE2ϕ. Then<br />

we haveADB2ϕ¡β¡γ,BCD180¥¡α¡β,AED360¥¡BCD¡CDE<br />

180¥¡2ϕ α β, <strong>and</strong> finallyDAE180¥¡ADE¡AED2ϕ¡α¡β¡γ.<br />

2ϕ − α − β − γ<br />

2ϕ − α − β<br />

B<br />

C<br />

B<br />

C<br />

O<br />

M<br />

N<br />

D<br />

M<br />

N<br />

O<br />

D<br />

E<br />

E<br />

Let N be the midpoint of CD; thenDNO90¥DMO, hence points M, N lie on<br />

the circle with diameter OD. Now, if points O <strong>and</strong> M lie on the same side of CD, we have<br />

DMNDON1<br />

2DOCα; in the other case, we haveDMN180¥¡DONα;


55<br />

so, in both casesDMNα(see Figures). Next, since MN is a midline triangle CDE,<br />

we haveMDEDMNα <strong>and</strong>NDM2ϕ¡α.<br />

Now we apply the sine rule to the triangles ABD, ADE (twice), BCD <strong>and</strong> MND obtaining<br />

AB<br />

AE<br />

γ DE<br />

ADsinÔ2ϕ¡β¡γÕ<br />

sinÔ2ϕ¡αÕ,<br />

ADsinÔ2ϕ¡α¡βÕ,<br />

ADsinÔ2ϕ¡α¡β¡γÕ<br />

sinÔ2ϕ¡α¡βÕ,<br />

BC β<br />

CDsin α ,<br />

α<br />

DECDß2<br />

DEß2ND<br />

NMsinÔ2ϕ¡αÕ,<br />

which implies<br />

β¤sinÔ2ϕ¡α¡β¡γÕ<br />

ADBC<br />

CD¤CD<br />

DE¤DE<br />

ADsin<br />

sinÔ2ϕ¡αÕ¤sinÔ2ϕ¡α¡βÕ.<br />

Hence, the condition ABAE BC, or equivalently<br />

ADAE AB BC<br />

, after multiplying<br />

AD<br />

by the common denominator rewrites as<br />

sinÔ2ϕ¡α¡βÕ¤sinÔ2ϕ¡β¡γÕsin γ¤sinÔ2ϕ¡αÕ sin β¤sinÔ2ϕ¡α¡β¡γÕ<br />

γ¡αÕ cosÔγ¡αÕ¡cosÔ4ϕ¡2β¡α¡γÕcosÔ2ϕ¡α¡2β¡γÕ¡cosÔ2ϕ<br />

cosÔγ¡αÕ cosÔ2ϕ γ¡αÕcosÔ2ϕ¡α¡2β¡γÕ cosÔ4ϕ¡2β¡α¡γÕ<br />

cosϕ¤cosÔϕ γ¡αÕcosϕ¤cosÔ3ϕ¡2β¡α¡γÕ<br />

cosÔϕ γ¡αÕ¡cosÔ3ϕ¡2β¡α¡γÕ¨0<br />

cosϕ¤sinÔ2ϕ¡β¡αÕ¤sinÔϕ¡β¡γÕ0.<br />

Since 2ϕ¡β¡α180¥¡AED180¥<strong>and</strong> ϕ1<br />

2ABC90¥, it follows that ϕβ γ,<br />

henceBDA2ϕ¡β¡γϕ1<br />

2CDE, as desired.


56<br />

G6. The vertices X, Y , Z of an equilateral triangle XY Z lie respectively on the sides BC,<br />

CA, AB of an acute-angled triangle ABC. Prove that the incenter of triangle ABC lies inside<br />

triangle XY Z.<br />

G6½.<br />

The vertices X, Y , Z of an equilateral triangle XY Z lie respectively on the sides<br />

BC, CA, AB of a triangle ABC. Prove that if the incenter of triangle ABC lies outside<br />

triangle XY Z, then one of the angles of triangle ABC is greater than 120¥.<br />

(Bulgaria)<br />

Solution 1 for G6. We will prove a stronger fact; namely, we will show that the incenter I of<br />

triangle ABC lies inside the incircle of triangle XY Z (<strong>and</strong> hence surely inside triangle XY Z<br />

itself). We denote by dÔU, V WÕthe distance between point U <strong>and</strong> line V W.<br />

Denote by O the incenter of △XY Z <strong>and</strong> by r, r½<strong>and</strong> R½the inradii of triangles ABC, XY Z<br />

<strong>and</strong> the circumradius of XY Z, respectively. Then we have R½2r½, <strong>and</strong> the desired inequality<br />

is OIr½. We assume that OI; otherwise the claim is trivial.<br />

Let the incircle of △ABC touch its sides BC, AC, AB at points A 1 , B 1 , C 1 respectively.<br />

The lines IA 1 , IB 1 , IC 1 cut the plane into 6 acute angles, each one containing one of the<br />

points A 1 , B 1 , C 1<br />

BCÕ<br />

on its border. We may assume that O lies in an angle defined by lines IA 1 ,<br />

IC 1 <strong>and</strong> containing point C 1 (see Fig. 1). Let A½<strong>and</strong> C½be the projections of O onto lines IA 1<br />

<strong>and</strong> IC 1 , respectively.<br />

Since OXR½, we have dÔO, BCÕR½. Since OA½‖ BC, it follows that dÔA½,<br />

A½I rR½, or A½IR½¡r. On the other h<strong>and</strong>, the incircle of △XY Z lies inside △ABC,<br />

hence dÔO, ABÕr½, <strong>and</strong> analogously we get dÔO, ABÕC½C 1r¡IC½r½, or IC½r¡r½.<br />

B<br />

X<br />

C 1<br />

C ′<br />

Z<br />

O C′<br />

I<br />

A ′<br />

O<br />

I<br />

A<br />

B 1 Y C<br />

A ′<br />

ÔR½¡rÕ Fig. 1 Fig. 2<br />

Finally, the quadrilateral IA½OC½is circumscribed due to the right angles at A½<strong>and</strong><br />

(see Fig. 2). On its circumcircle, we haveǑA½OC½2A½IC½180¥OC½I, hence 180¥<br />

IC½A½O. This means that IC½A½O. Finally, we have OIIA½ A½OIA½ IC½ Ôr¡r½ÕR½¡r½r½, as desired.<br />

Solution 2 for G6. Assume the contrary. Then the incenter I should lie in one of triangles<br />

AY Z, BXZ, CXY — assume that it lies in △AY Z. Let the incircle ω of △ABC touch<br />

sides BC, AC at point A 1 , B 1 respectively. Without loss of generality, assume that point A 1<br />

lies on segment CX. In this case we will show thatC90¥thus leading to a contradiction.<br />

Note that ω intersects each of the segments XY <strong>and</strong> Y Z at two points; let U, U½<strong>and</strong> V ,<br />

V½be the points of intersection of ω with XY <strong>and</strong> Y Z, respectively (UYU½Y , V YV½Y ;<br />

see Figs. 3 <strong>and</strong> 4). Note that 60¥XY Z1<br />

2ÔUV¡U½V½Õ1<br />

2UV , henceUV120¥.<br />

A 1<br />

C ′


On the other h<strong>and</strong>, since I lies in △AY Z, we getǑV UV½180¥, henceǑUA 1 U½ǑUA 180¥¡UV60¥.<br />

Now, two cases are possible due to the order of points Y , B 1 on segment AC.<br />

1<br />

V½<br />

57<br />

A<br />

ω C 1<br />

B 1<br />

C 1<br />

Y<br />

V ′<br />

Y<br />

I<br />

V ′<br />

I<br />

U ′<br />

V<br />

U ′<br />

B<br />

Z<br />

1 V<br />

Z<br />

U<br />

ω<br />

U<br />

C A 1 X<br />

B C A 1 X B<br />

Fig. 3 Fig. 4<br />

Case 1. Let point Y lie on the segment AB 1 (see Fig. 3). Then we haveY XC<br />

1A 2 1 U½¡A U¨1<br />

1 2ǑUA 1 U½30¥; analogously, we getXY C1<br />

2ǑUA 1<br />

X<br />

U½30¥. Therefore,<br />

Y CX180¥¡Y XC¡XY C120¥, as desired.<br />

Case 2. Now let point Y lie on the segment CB 1 (see Fig. 4). Analogously, we obtain<br />

Y XC30¥. Next,IY XZY X60¥, butIY XIY B 1 , since Y B 1 is a tangent<br />

<strong>and</strong> Y X is a secant line to circle ω from point Y . Hence, we get 120¥IY B 1 IY<br />

B 1 Y XY XC CX30¥ Y Y CX, henceY CX120¥¡30¥90¥, as desired.<br />

Comment. In the same way, one can prove a more general<br />

Claim. Let the vertices X, Y , Z of a triangle XY Z lie respectively on the sides BC, CA, AB of a<br />

triangle ABC. Suppose that the incenter of triangle ABC lies outside triangle XY Z, <strong>and</strong> α is the<br />

least angle of △XY Z. Then one of the angles of triangle ABC is greater than 3α¡90¥.<br />

A<br />

Solution for G6½. Assume the contrary. As in Solution 2, we assume that the incenter I of<br />

△ABC lies in △AY Z, <strong>and</strong> the tangency point A 1 of ω <strong>and</strong> BC lies on segment CX. Surely,<br />

Y ZA180¥¡Y ZX120¥, hence points I <strong>and</strong> Y lie on one side of the perpendicular<br />

bisector to XY ; therefore IXIY . Moreover, ω intersects segment XY at two points, <strong>and</strong><br />

therefore the projection M of I onto XY lies on the segment XY . In this case, we will prove<br />

thatC120¥.<br />

Let Y K, Y L be two tangents from point Y to ω (points K <strong>and</strong> A 1 lie on one side of XY ;<br />

if Y lies on ω, we say KLY ); one of the points K <strong>and</strong> L is in fact a tangency point B 1<br />

of ω <strong>and</strong> AC. From symmetry, we haveY IKY IL. On the other h<strong>and</strong>, since IXIY ,<br />

we get XMXY which impliesA 1<br />

MIKÕ<br />

XYKY X.<br />

Next, we haveMIY90¥¡IY X90¥¡ZY X30¥. Since IA 1ÃA 1 X, IMÃXY ,<br />

IKÃY K we getMIA 1A 1 XYKY XMIK. Finally, we get<br />

A 1 IKA 1 IM ÔKIY ILÕ Y<br />

2MIK 2KIY2MIY60¥.<br />

1 ILÔA<br />

Hence,A 1 IB 160¥, <strong>and</strong> thereforeACB180¥¡A 1 IB 1120¥, as desired.


58<br />

X A 1 K(= B 1 )<br />

Y<br />

M<br />

C<br />

A 1 M Y<br />

L(= B 1 )<br />

X<br />

I<br />

B<br />

Z<br />

A<br />

I<br />

Fig. 5 Fig. 6<br />

Comment 1. The estimate claimed in G6½is sharp. Actually, ifBAC120¥, one can consider an<br />

equilateral triangle XY Z with ZA, YÈAC, XÈBC (such triangle exists sinceACB60¥). It<br />

intersects with the angle bisector ofBAC only at point A, hence it does not contain I.<br />

Comment 2. As in the previous solution, there is a generalization for an arbitrary triangle XY Z,<br />

but here we need some additional condition. The statement reads as follows.<br />

Claim. Let the vertices X, Y , Z of a triangle XY Z lie respectively on the sides BC, CA, AB of a<br />

triangle ABC. Suppose that the incenter of triangle ABC lies outside triangle XY Z, α is the least<br />

angle of △XY Z, <strong>and</strong> all sides of triangle XY Z are greater than 2r cot α, where r is the inradius<br />

of △ABC. Then one of the angles of triangle ABC is greater than 2α.<br />

The additional condition is needed to verify that XMY M since it cannot be shown in the<br />

original way. Actually, we haveMY Iα, IMr, hence Y Mr cot α. Now, if we have<br />

XYXM YM2r cot α, then surely XMY M.<br />

On the other h<strong>and</strong>, this additional condition follows easily from the conditions of the original<br />

problem. Actually, if IÈ△AY Z, then the diameter of ω parallel to Y Z is contained in △AY Z <strong>and</strong><br />

is thus shorter than Y Z. Hence Y Z2r2r cot 60¥.


60<br />

G7. Three circular arcs γ 1 , γ 2 , <strong>and</strong> γ 3 connect the points A <strong>and</strong> C. These arcs lie in the same<br />

half-plane defined by line AC in such a way that arc γ 2 lies between the arcs γ 1 <strong>and</strong> γ 3 . Point<br />

B lies on the segment AC. Let h 1 , h 2 , <strong>and</strong> h 3 be three rays starting at B, lying in the same<br />

half-plane, h 2 being between h 1 <strong>and</strong> h 3 . For i, j1, 2, 3, denote by V ij the point of intersection<br />

of h i <strong>and</strong> γ j (see the Figure below).<br />

Denote byǑV ij V kjǑV kl V il the curved quadrilateral, whose sides are the segments V ij V il , V kj V kl<br />

<strong>and</strong> arcs V ij V kj <strong>and</strong> V il V kl . We say that this quadrilateral is circumscribed if there exists a circle<br />

touching these two segments <strong>and</strong> two arcs.<br />

Prove that if the curved quadrilateralsǑV 11 V 21ǑV 22 V 12 ,ǑV 12 V 22ǑV 23 V 13 ,ǑV 21 V 31ǑV 32 V 22 are circumscribed,<br />

then the curved quadrilateralǑV 22 V 32ǑV 33 V 23 is circumscribed, too.<br />

h<br />

V 2 23<br />

h 1<br />

V 22<br />

γ 3<br />

V 13<br />

V 33<br />

V 12<br />

V 11<br />

V 32<br />

h 3<br />

A<br />

γ 2<br />

V 21 V 31<br />

γ 1<br />

B<br />

Fig. 1<br />

C<br />

(Hungary)<br />

Solution. Denote by O i <strong>and</strong> R i the center <strong>and</strong> the radius of γ i , respectively. Denote also by H<br />

the half-plane defined by AC which contains the whole configuration. For every point P in<br />

the half-plane H, denote by dÔPÕthe distance between P <strong>and</strong> line AC. Furthermore, for any<br />

r0, denote by ΩÔP, rÕthe circle with center P <strong>and</strong> radius r.<br />

Lemma 1. For every 1ij3, consider those circles ΩÔP, rÕin the half-plane H which<br />

are tangent to h i <strong>and</strong> h j .<br />

(a) The locus of the centers of these circles is the angle bisector β ij between h i <strong>and</strong> h j .<br />

(b) There is a constant u ij such that ru ij¤dÔPÕfor<br />

Ð<br />

all such circles.<br />

Proof. Part (a) is obvious. To prove part (b), notice that the circles which are tangent to h i<br />

<strong>and</strong> h j are homothetic with the common homothety center B (see Fig. 2). Then part (b) also<br />

becomes trivial.<br />

Lemma 2. For every 1ij3, consider those circles ΩÔP, rÕin the half-plane H which<br />

are externally tangent to γ i <strong>and</strong> internally tangent to γ j .<br />

(a) The locus of the centers of these circles is an ellipse arc ε ij with end-points A <strong>and</strong> C.<br />

(b) There is a constant v ij such that rv ij¤dÔPÕfor all such circles.<br />

Proof. (a) Notice that the circle ΩÔP, rÕis externally tangent to γ i <strong>and</strong> internally tangent to γ j<br />

if <strong>and</strong> only if O i PR i r <strong>and</strong> O jR j¡r. Therefore, for each such circle we have<br />

O i P O j PO i A O j AO i C O j CR i R j .<br />

Such points lie on an ellipse with foci O i <strong>and</strong> O j ; the diameter of this ellipse is R i R j , <strong>and</strong> it<br />

passes through the points A <strong>and</strong> C. Let ε ij be that arc AC of the ellipse which runs inside the<br />

half plane H (see Fig. 3.)<br />

This ellipse arc lies between the arcs γ i <strong>and</strong> γ j . Therefore, if some point P lies on ε ij ,<br />

then O i PR i <strong>and</strong> O j PR j . Now, we choose rO i P¡R iR j¡O j P0; then the


61<br />

Ω(P, r)<br />

γ j<br />

h i<br />

β ij<br />

r<br />

P<br />

R j<br />

ε ij<br />

P<br />

r<br />

P ′<br />

r ′<br />

d(P ′ )<br />

h j<br />

A<br />

d(P)<br />

B<br />

⃗ρ<br />

Fig. 2 Fig. 3<br />

r, i j<br />

circle ΩÔP, rÕtouches γ i externally <strong>and</strong> touches γ j<br />

,P<br />

internally, so P belongs to the locus under<br />

investigation.<br />

(b) Let AP, ⃗ρ AO i , <strong>and</strong> ⃗ρ AO j ; let d ijO i O j , <strong>and</strong> let ⃗v be a unit vector<br />

orthogonal to AC <strong>and</strong> directed toward H. Then we have⃗ρ iR i ,⃗ρ jR j O i<br />

⃗ρ¡⃗ρ iR i O j<br />

jÕ P⃗ρ¡⃗ρ jR j¡r,<br />

jÕ<br />

hence<br />

Ô⃗ρ¡⃗ρ iÕ2¡Ô⃗ρ¡⃗ρ jÕ2ÔR i rÕ2¡ÔR j¡rÕ2 ,<br />

Ô⃗ρ i¡⃗ρ 2 2 2⃗ρ¤Ô⃗ρ j¡⃗ρ iÕÔR i¡R 2 2 2rÔR i R jÕ,<br />

d ij¤dÔPÕd ij<br />

r<br />

⃗v¤⃗ρÔ⃗ρ j¡⃗ρ iÕ¤⃗ρrÔR i R jÕ.<br />

Therefore,<br />

d R i R j¤dÔPÕ,<br />

<strong>and</strong> the value v ij<br />

d<br />

ijǑ Ǒ Ð<br />

ij<br />

does not depend on P.<br />

R i R j<br />

Lemma 3. The curved quadrilateral Q V i,j V i 1,jV i 1,j 1V i,j 1 is circumscribed if <strong>and</strong> only<br />

if u i,i 1v j,j 1.<br />

Proof. First suppose that the curved quadrilateral Q ij is circumscribed <strong>and</strong> ΩÔP, rÕis its inscribed<br />

circle. By Lemma 1 <strong>and</strong> Lemma 2 we have ru i,i 1¤dÔPÕ<strong>and</strong> rv j,j 1¤dÔPÕas<br />

well. Hence, u i,i 1v j,j 1.<br />

To prove the opposite direction, suppose u i,i 1v j,j<br />

Ð<br />

1. Let P be the intersection of the<br />

angle bisector β i,i 1 <strong>and</strong> the ellipse arc ε j,j 1. Choose ru i,i 1¤dÔPÕv j,j 1¤dÔPÕ. Then<br />

the circle ΩÔP, rÕis tangent to the half lines h i <strong>and</strong> h i 1 by Lemma 1, <strong>and</strong> it is tangent to the<br />

arcs γ j <strong>and</strong> γ j 1 by Lemma 2. Hence, the curved quadrilateral Q ij is circumscribed.<br />

By Lemma 3, the statement of the problem can be reformulated to an obvious fact: If the<br />

equalities u 12v 12 , u 12v 23 , <strong>and</strong> u 23v 12 hold, then u 23v 23 holds as well.<br />

⃗ρ⃗ρ⃗ρ⃗ρ⃗ρ⃗ρ⃗ρ⃗ρ⃗ρ⃗ρ⃗ρ⃗ρ⃗ρ⃗ρ⃗ρ⃗ρ⃗ρ⃗ρ⃗ρ⃗ρ⃗ρ⃗ρ⃗ρ⃗ρ⃗ρ⃗ρ⃗ρ⃗ρ⃗ρ⃗ρ⃗ρ⃗ρ⃗ρ⃗ρ⃗ρ⃗ρ⃗ρ⃗ρ⃗ρ⃗ρ⃗ρ⃗ρ⃗ρ⃗ρ⃗ρ⃗ρ⃗ρ⃗ρ⃗ρ⃗ρ⃗ρ⃗ρ⃗ρ⃗ρ⃗ρ⃗ρ⃗ρ⃗ρ⃗ρ⃗ρ⃗ρ⃗ρ⃗ρ⃗ρ⃗ρ<br />

r<br />

⃗ρ j<br />

⃗ρ i<br />

O j<br />

O i<br />

R i<br />

γ i<br />

⃗v<br />

C


62<br />

Comment 1. Lemma 2(b) (together with the easy Lemma 1(b)) is the key tool in this solution.<br />

If one finds this fact, then the solution can be finished in many ways. That is, one can find a circle<br />

touching three of h 2 , h 3 , γ 2 , <strong>and</strong> γ 3 , <strong>and</strong> then prove that it is tangent to the fourth one in either<br />

synthetic or analytical way. Both approaches can be successful.<br />

Here we present some discussion about this key Lemma.<br />

1. In the solution above we chose an analytic proof for Lemma 2(b) because we expect that most<br />

students will use coordinates or vectors to examine the locus of the centers, <strong>and</strong> these approaches are<br />

less case-sensitive.<br />

Here we outline a synthetic proof. We consider only the case when P does not lie in the line O i O j .<br />

The other case can be obtained as a limit case, or computed in a direct way.<br />

Let S be the internal homothety center between the circles of γ i <strong>and</strong> γ j , lying on O i O j ; this point<br />

does not depend on P. Let U <strong>and</strong> V be the points of tangency of circle σΩÔP,rÕwith γ i <strong>and</strong> γ j ,<br />

respectively (then rPUPV ); in other words, points U <strong>and</strong> V are the intersection points of<br />

rays O i P, O j P with arcs γ i , γ j respectively (see Fig. 4).<br />

Due to the theorem on three homothety centers (or just to the Menelaus theorem applied to<br />

triangle O i O j<br />

P<br />

P), the points U, V <strong>and</strong> S are collinear. Let T be the intersection point of line AC <strong>and</strong><br />

the common tangent to σ <strong>and</strong> γ i at U; then T is the radical center of σ, γ i <strong>and</strong> γ j , hence TV is the<br />

common tangent to σ <strong>and</strong> γ j<br />

Ð<br />

.<br />

Let Q be the projection of P onto the line AC. By the right angles, the points U, V <strong>and</strong> Q lie on<br />

the circle with diameter PT. From this fact <strong>and</strong> the equality PUPV we getUQPUV<br />

V UPSUO i . Since O i S ‖ PQ, we haveSO i UQPU. Hence, the triangles SO i U <strong>and</strong> UPQ<br />

r<br />

i S i S<br />

are similar <strong>and</strong> thus<br />

; the last expression is constant since S is a constant<br />

dÔPÕPU<br />

PQO O i UO R i<br />

point.<br />

l<br />

V<br />

P<br />

σ<br />

γ j<br />

d l (A)<br />

P<br />

d l (P)<br />

O j<br />

ε ij<br />

U<br />

γ i<br />

d(P)<br />

O j<br />

S<br />

A<br />

C<br />

T<br />

A<br />

Q<br />

C<br />

O i<br />

O i<br />

Fig. 4 Fig. 5<br />

2. Using some known facts about conics, the same statement can be proved in a very short way.<br />

Denote by l the directrix of ellipse of ε ij related to the focus O j ; since ε ij is symmetrical about O i O j ,<br />

we have l ‖ AC. Recall that for each point PÈε ij , we have PO jǫ¤d lÔPÕ, where d lÔPÕis the<br />

distance from P to l, <strong>and</strong> ǫ is the eccentricity of ε ij<br />

Ð<br />

(see Fig. 5).<br />

Now we have<br />

rR j¡ÔR j¡rÕAO j¡PO jǫ d lÔAÕ¡d lÔPÕ¨ǫ dÔPÕ¡dÔAÕ¨ǫ¤dÔPÕ,<br />

<strong>and</strong> ǫ does not depend on P.


63<br />

Comment 2. One can find a spatial interpretations of the problem <strong>and</strong> the solution.<br />

For every pointÔx,yÕ<strong>and</strong> radius r0, represent the circle Ω Ôx,yÕ,r¨by the pointÔx,y,rÕ<br />

in space. This point is the apex of the cone with base circle Ω Ôx,yÕ,r¨<strong>and</strong> height r. According to<br />

Lemma 1, the circles which are tangent to h i <strong>and</strong> h j correspond to the points of a half line β½ij , starting<br />

at B.<br />

Now we translate Lemma 2. Take some 1ij3, <strong>and</strong> consider those circles which are<br />

internally tangent to γ j . It is easy to see that the locus of the points which represent these circles is<br />

a subset of a cone, containing γ j . Similarly, the circles which are externally tangent to γ i correspond<br />

to the points on the extension of another cone, which has its apex on the opposite side of the base<br />

plane Π. (See Fig. 6; for this illustration, the z-coordinates were multiplied by 2.)<br />

The two cones are symmetric to each other (they have the same aperture, <strong>and</strong> their axes are<br />

parallel). As is well-known, it follows that the common points of the two cones are co-planar. So the<br />

intersection of the two cones is a a conic section — which is an ellipse, according to Lemma 2(a). The<br />

points which represent the circles touching γ i <strong>and</strong> γ j is an ellipse arc ε½ij with end-points A <strong>and</strong> C.<br />

β 12<br />

′<br />

ε ′ ij<br />

γ i<br />

γ j<br />

Π<br />

Σ<br />

ε ′ 23<br />

ε ′ 12<br />

β ′ 23<br />

Fig. 6 Fig. 7<br />

Thus, the curved quadrilateral Q ij is circumscribed if <strong>and</strong> only if β½i,i 1 <strong>and</strong> ε½j,j 1 intersect, i.e. if<br />

they are coplanar. If three of the four curved quadrilaterals are circumscribed, it means that ε½12 , ε½23 ,<br />

<strong>and</strong> β½23 lie in the same plane Σ, <strong>and</strong> the fourth intersection comes to existence, too (see Fig. 7).<br />

β½12<br />

A connection between mathematics <strong>and</strong> real life:<br />

the Palace of Creativity “Shabyt” (“Inspiration”) in Astana


Number Theory<br />

N1. Find the least positive integer n for which there exists a setØs 1 , s 2 , . . .,s nÙconsisting of<br />

n distinct positive integers such that<br />

¢1¡1<br />

1Å¢1¡1<br />

s s 2Å...¢1¡1<br />

s nÅ51<br />

2010 .<br />

N1½. Same as Problem N1, but the constant<br />

51<br />

2010<br />

is replaced by<br />

42<br />

2010 . (Canada)<br />

Answer for Problem N1. n39.<br />

Solution for Problem N1. Suppose that for some n there exist the desired numbers; we<br />

may assume that s 1s 2¤¤¤s n . Surely s 11since<br />

nÅ<br />

otherwise 1¡1<br />

So we have<br />

s 10.<br />

2s 1s 2¡1¤¤¤s n¡Ôn¡1Õ, hence s ii 1 for each i1, . . .,n. Therefore<br />

51<br />

2010¢1¡1<br />

1Å¢1¡1<br />

s s 2Å...¢1¡1<br />

1<br />

s<br />

¢1¡1 2Å¢1¡1<br />

3Å...¢1¡1<br />

1Å1<br />

n 2¤2<br />

3¤¤¤n 1<br />

n n 1 ,<br />

which implies<br />

n 12010<br />

51670<br />

1739,<br />

so n39.<br />

Now we are left to show that n39 fits. Consider the setØ2, 3, . . ., 33, 35, 36, . . ., 40, 67Ù<br />

Ô1Õ<br />

which contains exactly 39 numbers. We have<br />

1<br />

2¤2<br />

3¤¤¤32<br />

33¤34<br />

35¤¤¤39<br />

40¤66<br />

671<br />

33¤34<br />

40¤66<br />

6717<br />

67051<br />

2010 ,<br />

hence for n39 there exists a desired example.<br />

Comment. One can show that the exampleÔ1Õis unique.<br />

Answer for Problem N1½. n48.<br />

Solution for Problem N1½. Suppose that for some n there exist the desired numbers. In<br />

the same way we obtain that s ii 1. Moreover, since the denominator of the fraction<br />

42<br />

20107<br />

335 is divisible by 67, some of s i’s should be divisible by 67, so s ns i67. This<br />

3¤¤¤n¡1<br />

means that<br />

42<br />

20101<br />

2¤2<br />

n¤¢1¡1<br />

67Å66<br />

67n ,


65<br />

which implies<br />

n2010¤66<br />

42¤67330<br />

747,<br />

so n48.<br />

Now we are left to show that n48 fits. Consider the setØ2, 3, . . ., 33, 36, 37, . . ., 50, 67Ù<br />

which contains exactly 48 numbers. We have<br />

1<br />

2¤2<br />

3¤¤¤32<br />

33¤35<br />

36¤¤¤49<br />

50¤66<br />

671<br />

33¤35<br />

50¤66<br />

677<br />

33542<br />

2010 ,<br />

hence for n48 there exists a desired example.<br />

Comment 1. In this version of the problem, the estimate needs one more step, hence it is a bit<br />

harder. On the other h<strong>and</strong>, the example in this version is not unique. Another example is<br />

1<br />

2¤2<br />

3¤¤¤46<br />

47¤66<br />

67¤329<br />

3301<br />

67¤66<br />

330¤329<br />

477<br />

67¤542<br />

2010 .<br />

Comment 2. N1½was the Proposer’s formulation of the problem. We propose N1 according to the<br />

number of current IMO.


66<br />

N2. Find all pairsÔm, nÕof nonnegative integers for which<br />

m 2 2¤3 nm 2 n 1¡1¨. (1)<br />

Answer.Ô6, 3Õ,Ô9, 3Õ,Ô9, 5Õ,Ô54, 5Õ.<br />

(Australia)<br />

Solution. For fixed values of n, the equation (1) is a simple quadratic equation in m. For<br />

n5 the solutions are listed in the following table.<br />

case equation discriminant integer roots<br />

n0 m 2¡m 20 ¡7 none<br />

n1 m 2¡3m 60 ¡15 none<br />

n2 m 2¡7m 180 ¡23 none<br />

n3 m 2¡15m 540 9 m6 <strong>and</strong> m9<br />

n4 m 2¡31m 1620 313 none<br />

n5 m 2¡63m 4860 202545 2 m9 <strong>and</strong> m54<br />

We prove that there is no solution for n6.<br />

Suppose thatÔm, nÕsatisfies (1) <strong>and</strong> n6. Since m§2¤3nm 2 n 1¡1¨¡m<br />

m3p with some 0pn or m2¤3q with some 0qn.<br />

In the first case, let qn¡p; then<br />

In the second case let pn¡q. Then<br />

2 n 1¡1m 2¤3 n<br />

m3 p 2¤3 q .<br />

2 , we have<br />

2 n 2¤3 1¡1m n<br />

m2¤3 q 3 p .<br />

1Õ<br />

Hence, in both cases we need to find the nonnegative integer solutions of<br />

3 p 2¤3 q2 n 1¡1, p qn. (2)<br />

Next, we prove<br />

1Õ<br />

1Õ<br />

bounds for p, q. From (2) we get<br />

3 p2 n 18 n 1<br />

39 n 1<br />

33 1Õ<br />

2Ôn 3<br />

<strong>and</strong><br />

2¤3 q2 n 12¤8 n 32¤9 n 32¤3 2n 32¤3 2Ôn 3 ,<br />

so p, q2Ôn . Combining these inequalities with p qn, we obtain<br />

3<br />

n¡2<br />

q2Ôn p, . (3)<br />

3 3<br />

Now let hminÔp, qÕ. By (3) we have hn¡2<br />

3 ; in particular, we have h1. On the<br />

left-h<strong>and</strong> side of (2), both terms are divisible by 3 h , therefore 9§3 h§2 n 1¡1. It is easy check<br />

that ord 9Ô2Õ6, so 9§2 n 1¡1 if <strong>and</strong> only if 6§n 1. Therefore, n 16r for some positive<br />

integer r, <strong>and</strong> we can write<br />

2 n 1¡14 3r¡1Ô4 2r 4 r 1ÕÔ2<br />

r¡1ÕÔ2 r 1Õ. (4)


Notice that the factor 4 2r 4 r 1Ô4 divisible by 9. The other two factors in (4), 2 r¡1 <strong>and</strong> 2 r 1 are coprime: both are odd <strong>and</strong><br />

67<br />

r¡1Õ2 3¤4r is divisible by 3, but it is never<br />

their difference is 2. Since the whole product is divisible by 3 h , we have either 3 h¡1§2 r¡1 or<br />

3 h¡1§2 r 1. In any case, we have 3 h¡12 r 1. Then<br />

3 h¡12 r 13 r3 n 1<br />

6 ,<br />

n¡2<br />

1<br />

,<br />

3¡1h¡1n 6<br />

n11.<br />

But this is impossible since we assumed n6, <strong>and</strong> we proved 6§n 1.


¤¤¤<br />

68<br />

N3. Find the smallest number n such that there exist polynomials f 1 , f 2 , ..., f n with rational<br />

coefficients satisfying<br />

x 2 7f 1ÔxÕ2 f 2ÔxÕ2 f nÔxÕ2 .<br />

(Pol<strong>and</strong>)<br />

Answer. The smallest n is 5.<br />

Solution 1. The equality x 2 7x2 2 2 1 2 1 2 1 2 shows that n5. It remains to<br />

show that x 2 7 is not a sum of four (or less) squares of polynomials with rational coefficients.<br />

Suppose by way of contradiction that x 2 7f 1ÔxÕ2 f 2ÔxÕ2 f 3ÔxÕ2 f 4ÔxÕ2 , where the<br />

coefficients of polynomials f 1 , f 2 , f 3 <strong>and</strong> f 4 are rational (some of these polynomials may be<br />

zero).<br />

Clearly, the degrees of f 1 , f 2 , f 3 <strong>and</strong> f 4 are at most 1. Thus f iÔxÕa i x b i for i1, 2, 3, 4<br />

<strong>and</strong> some rationals a 1 , b 1 , a 2 , b 2 , a 3 , b 3 , a 4 , b 4 . It follows that x 2 74<br />

i1Ôa i x b iÕ2 <strong>and</strong><br />

hence<br />

4ôi1<br />

a 2 i1,<br />

4ôi1<br />

a i b i0,<br />

Let p ia i b i <strong>and</strong> q ia i¡b i for i1, 2, 3, 4. Then<br />

4ôi1<br />

b 2 i7. (1)<br />

4ôi1<br />

p 2 i4ôi1 a 2 i 2<br />

4ôi1<br />

a i b i<br />

4ôi1b 2 i8,<br />

4ôi1<br />

q 2 i4ôi1 a 2 i¡2<br />

4ôi1<br />

a i b i<br />

4ôi1b 2 i8<br />

<strong>and</strong><br />

4ôi1<br />

p i q i4ôi1<br />

a 2 i¡4ôi1 b 2 i¡6,<br />

which means that there exist a solution in integers x 1 , y 1 , x 2 , y 2 , x 3 , y 3 , x 4 , y 4 <strong>and</strong> m0of<br />

the system of equations<br />

4ôi1<br />

4ôi1<br />

4ôi1<br />

(i)<br />

x 2 i8m 2 ,<br />

(ii)<br />

y 2 i8m 2 ,<br />

(iii)<br />

x i y i¡6m 2 .<br />

We will show that such a solution does not exist.<br />

Assume the contrary <strong>and</strong> consider a solution with minimal m. Note that if an integer x is<br />

odd then x 21Ômod 8Õ. Otherwise (i.e., if x is even) we have x 20Ômod 8Õor x 24<br />

Ômod 8Õ. Hence, by (i), we get that x 1 , x 2 , x 3 <strong>and</strong> x 4 are even. Similarly, by (ii), we get that<br />

y 1 , y 2 , y 3 <strong>and</strong> y 4 are even. Thus the LHS of (iii) is divisible by 4 <strong>and</strong> m is also even. It follows<br />

thatÔx 1<br />

2<br />

, y 1<br />

, x 2<br />

2 2<br />

, y 2<br />

, x 3<br />

2 2<br />

, y 3<br />

, x 4<br />

2 2<br />

, y 4<br />

, m 2 2Õis a solution of the system of equations (i), (ii) <strong>and</strong> (iii),<br />

which contradicts the minimality of m.<br />

Solution 2. We prove that n4 is impossible. Define the numbers a i , b i for i1, 2, 3, 4 as<br />

in the previous solution.<br />

By Euler’s identity we have<br />

Ôa 2 1 a 2 2 a 2 3 a 2 4ÕÔb 2 1 b 2 2 b 2 3 b 2 4ÕÔa<br />

1<br />

b 1 a 2 b 2 a 3 b 3 a 4 b 4Õ2 Ôa 1 b 3¡a 3 b 1 a 4 b 2¡a 2 b 4Õ2<br />

Ôa 1 b 2¡a 2 b 1 a 3 b 4¡a 4 b 3Õ2<br />

Ôa 1 b 4¡a 4 b 1 a 2 b 3¡a 3 b 2Õ2 .


69<br />

So, using the relations (1) from the Solution 1 we get that<br />

where<br />

7¡m 1<br />

m©2 ¡m 2<br />

m©2 ¡m 3<br />

m©2<br />

, (2)<br />

m 1<br />

1 b 2¡a 2 b 1 a 3 b 4¡a 4 b 3 ,<br />

ma m 2<br />

1 b 3¡a 3 b 1 a 4 b 2¡a 2 b 4 ,<br />

ma m 3<br />

1 b 4¡a 4 b 1 a 2 b 3¡a 3 b 2<br />

ma <strong>and</strong> m 1 , m 2 , m 3ÈZ,<br />

2¨<br />

mÈN.<br />

Let m be a minimum positive integer number for which (2) holds. Then<br />

8m 2m 2 1 m 2 2 m 2 3 m 2 .<br />

m<br />

As in the previous solution, we get that m 1 , m 2 , m 3 , m are all even numbers. Then 1<br />

, m 2<br />

, m 3<br />

, m<br />

2 2 2<br />

is also a solution of (2) which contradicts the minimality of m. So, we have n5. The example<br />

with n5 is already shown in Solution 1.


70<br />

N4. Let a, b be integers, <strong>and</strong> let PÔxÕax 3 bx. For any positive integer n we say that the<br />

pairÔa, bÕis n-good if n§PÔmÕ¡PÔkÕimplies n§m¡k for all integers m, k. We say that<br />

Ôa, bÕis very good ifÔa, bÕis n-good for infinitely many positive integers n.<br />

(a) Find a pairÔa, bÕwhich is 51-good, but not very good.<br />

(b) Show that all 2010-good pairs are very good.<br />

(Turkey)<br />

Solution. (a) We show that the pairÔ1,¡51 2Õis good but not very good. Let PÔxÕx3¡512 x.<br />

Since PÔ51ÕPÔ0Õ, the pairÔ1,¡51 2Õis not n-good for any positive integer that does not<br />

divide 51. Therefore,Ô1,¡51 2Õis not very good.<br />

On the other h<strong>and</strong>, if PÔmÕPÔkÕÔmod 51Õ, then m 3k3Ômod 51Õ. By Fermat’s<br />

theorem, from this we obtain<br />

mm 3k 3kÔmod 3Õ<strong>and</strong> mm 33k 33kÔmod 17Õ.<br />

Hence we have mkÔmod 51Õ. ThereforeÔ1,¡51 2Õis 51-good.<br />

PÔm½ÕPÔmÕPÔkÕPÔk½Õ<br />

(b) We will show that if a pairÔa, bÕis 2010-good thenÔa, bÕis 67 i -good for all positive<br />

integer i.<br />

Ð<br />

Claim 1. IfÔa, bÕis 2010-good thenÔa, bÕis 67-good.<br />

Proof. Assume that PÔmÕPÔkÕÔmod 67Õ. Since 67 <strong>and</strong> 30 are coprime, there exist integers<br />

m½<strong>and</strong> k½such that k½kÔmod 67Õ, k½0Ômod 30Õ, <strong>and</strong> m½mÔmod 67Õ, m½0<br />

Ômod 30Õ. Then we have PÔm½ÕPÔ0ÕPÔk½ÕÔmod 30Õ<strong>and</strong><br />

Ômod 67Õ, hence PÔm½ÕPÔk½ÕÔmod 2010Õ. This implies m½k½Ômod 2010ÕasÔa, bÕis<br />

2010-good. It follows that mm½k½kÔmod 67Õ. Therefore,Ôa, bÕis 67-good.<br />

Claim 2. IfÔa, bÕis 67-good then 3Õ 67§a.<br />

bÔm¡kÕÔm¡kÕ 2Õ b¨<br />

Proof. Suppose that 67Чa. Consider the setsØat 2Ômod 67Õ:0t33Ù<strong>and</strong>Ø¡3as 2¡b<br />

mod 67 : 0s33Ù. Since a0Ômod 67Õ, each of these sets has 34 elements. Hence they<br />

have at least one element in common. If at 2¡3as2¡bÔmod 67Õthen for mt¨s, k©2s<br />

we have<br />

PÔmÕ¡PÔkÕaÔm 3¡k aÔm 2 mk k<br />

Ôt¨3sÕÔat 2 3as 2 bÕ0Ômod 67Õ.<br />

SinceÔa, bÕis 67-good, we must have mkÔmod 67Õin both cases, that is, t3sÔmod 67Õ<br />

<strong>and</strong> t¡3sÔmod i§PÔmÕ¡PÔkÕÔm¡kÕ 67Õ. This means ts0Ômod 67Õ<strong>and</strong> b¡3as 2¡at20Ômod 67Õ.<br />

But then 67§PÔ7Õ¡PÔ2Õ67¤5a 2Õ 2Õ 5b <strong>and</strong> 67Ч7¡2, contradicting thatÔa, bÕis 67-good.Ð<br />

bÕ Ð<br />

Claim 3. IfÔa, bÕis 2010-good thenÔa, bÕis 67 i -good all i1.<br />

Proof. By Claim 2, we have 67§a. If 67§b, then PÔxÕPÔ0ÕÔmod 67Õfor all x, contradicting<br />

thatÔa, bÕis 67-good. Hence, 67Чb.<br />

Suppose that 67 aÔm2 mk k b¨. Since 67§a <strong>and</strong> 67Чb,<br />

the second factor aÔm2 mk k b is coprime to 67 <strong>and</strong> hence 67 i§m¡k. Therefore,Ôa,<br />

is 67 i -good.<br />

Comment 1. In the proof of Claim 2, the following reasoning can also be used. Since 3 is not<br />

a quadratic residue modulo 67, either au 2¡bÔmod 67Õor 3av 2¡bÔmod 67Õhas a solution.<br />

The settingsÔm,kÕÔu,0Õin the first case <strong>and</strong>Ôm,kÕÔv,¡2vÕin the second case lead to b0<br />

Ômod 67Õ.<br />

Comment 2. The pairÔ67,30Õis n-good if <strong>and</strong> only if nd¤67 i , where d§30 <strong>and</strong> i0. It shows<br />

that in part (b), one should deal with the large powers of 67 to reach the solution. The key property<br />

of number 67 is that it has the form 3k 1, so there exists a nontrivial cubic root of unity modulo 67.


fÔmÕ n¨<br />

71<br />

N5. Let N be the set of all positive integers. Find all functions f : NNsuch that the<br />

number m fÔnÕ¨is a square for all m,<br />

fÔmÕ n¨<br />

nÈN.<br />

fÔnÕ<br />

(U.S.A.)<br />

Answer. All functions of the form fÔnÕn c, where cÈNØ0Ù.<br />

Solution. First, it is clear that all functions of<br />

fÔlÕfÔkÕ<br />

the form fÔnÕn c with a constant nonnegative<br />

integer c satisfy the problem conditions since m¨Ôn m cÕ2 is a<br />

square.<br />

We are left to prove that there are no other functions. We start with the following<br />

Lemma. Suppose that p§fÔkÕ¡fÔlÕfor some prime p <strong>and</strong> positive integers k, l. Then p§k¡l.<br />

p§<br />

Proof.<br />

fÔnÕ fÔkÕ<br />

Suppose<br />

k¨¡ n¨<br />

first<br />

fÔnÕ fÔnÕ fÔlÕ<br />

that p 2§fÔkÕ¡fÔlÕ, fÔnÕ n¨ fÔnÕ fÔnÕ<br />

so p 2 a for some integer a. Take some<br />

positive integer DmaxØfÔkÕ, fÔlÕÙwhich is not divisible by p <strong>and</strong> set npD¡fÔkÕ. Then<br />

the positive numbers n fÔkÕpD <strong>and</strong> n fÔlÕpD fÔlÕ¡fÔkÕ¨pÔD paÕare<br />

both divisible by p but not by p 2 . Now, applying the problem conditions, we get that both the<br />

numbers k¨<strong>and</strong> l¨are squares divisible by p (<strong>and</strong> thus<br />

by p 2 ); this<br />

p§<br />

means<br />

fÔnÕ<br />

that<br />

k¨¡<br />

the<br />

fÔnÕ fÔnÕ fÔkÕ fÔnÕ fÔlÕ n<br />

multipliers k <strong>and</strong> l are also divisible by p, therefore<br />

Ð<br />

l¨k¡l as well.<br />

On the other h<strong>and</strong>, if fÔkÕ¡fÔlÕis divisible by p but not by p 2 , then choose the same<br />

number D <strong>and</strong> set np3 D¡fÔkÕ. Then the positive numbers np3 D <strong>and</strong><br />

p 3 D fÔlÕ¡fÔkÕ¨are respectively divisible by p 3 (but not by p 4 ) <strong>and</strong> by p (but not by p 2 ).<br />

Hence in analogous way we obtain that the numbers k <strong>and</strong> l are divisible by p,<br />

therefore l¨k¡l.<br />

We turn to the problem. First, suppose that fÔkÕfÔlÕfor some k, lÈN. Then by Lemma<br />

we<br />

fÔnÕ¨qfÔ1Õ fÔnÕfÔ1Õ<br />

have that k¡l is divisible by every prime number, so k¡l0, or kl. Therefore, the<br />

function f is injective.<br />

Next, consider the<br />

fÔnÕfÔ1Õ fÔnÕfÔn¡2ÕfÔ1Õ 1Õ<br />

numbers fÔkÕ<strong>and</strong> fÔk 1Õ. Since the numberÔk 1Õ¡k1has no<br />

prime divisors, by Lemma the same holds for fÔk 1Õ¡fÔkÕ; fÔnÕÔfÔ1Õ¡1Õ fÔnÕfÔ1Õ nfÔ1Õ<br />

thusfÔk 1Õ¡fÔkÕ1.<br />

Now, let fÔ2Õ¡fÔ1Õq,q1. Then we prove by induction that qÔn¡1Õ.<br />

The base for n1, 2 holds by the definition of q. For the step, if n1we have qÔn¡1Õ¨q. Since qÔn¡2Õ, we get qn,<br />

as desired.<br />

Finally, we have qÔn¡1Õ. Then q cannot be¡1 since otherwise for 1<br />

we have fÔnÕ0 which is impossible. Hence q1 <strong>and</strong> n for each nÈN,<br />

<strong>and</strong> fÔ1Õ¡10, as desired.


72<br />

N6. The rows <strong>and</strong> columns of a 2 n¢2 n table are numbered from 0 to 2 n¡1. The cells of the<br />

table have been colored with the following property being satisfied: for each 0i, j2 n¡1,<br />

the jth cell in the ith row <strong>and</strong> theÔi jÕth cell in the jth row have the same color. (The<br />

indices of the cells in a row are considered modulo 2 n .)<br />

Prove that the maximal possible number of colors is 2 n .<br />

jÕ<br />

(Iran)<br />

Solution. Throughout the solution we denote the cells of the table by coordinate pairs;Ôi,<br />

refers to the jth cell in the ith row.<br />

Consider the directed graph, whose vertices are the cells of the board, <strong>and</strong> the edges are<br />

the arrowsÔi, jÕÔj, i jÕfor all 0i, j2n¡1. From each vertexÔi, jÕ, exactly one edge<br />

passes (toÔj, i j mod 2 nÕ); conversely, to each cellÔj, kÕexactly one edge is directed (from<br />

the cellÔk¡j mod 2 n , jÕÕ. Hence, the graph splits into cycles.<br />

Now, in any coloring considered, the vertices of each cycle should have the same color by<br />

the problem condition. On the other h<strong>and</strong>, if each cycle has its own color, the obtained coloring<br />

obviously satisfies the problem conditions. Thus, the maximal possible number of colors is the<br />

same as the number of cycles, <strong>and</strong> we have to prove that this number is 2 n .<br />

Next, consider any cycleÔi 1 , j 1Õ,Ôi 2 , j 2Õ, . . .; we will describe it in other terms. Define a<br />

sequenceÔa 0 , a 1 , . . .Õby the relations a 0i 1 , a 1j 1 , a n 1a n a n¡1 2<br />

for all n1(we<br />

say that such a sequence is a Fibonacci-type sequence). Then an obvious induction shows<br />

that i ka k¡1Ômod 2 nÕ, j ka kÔmod 2 nÕ. Hence we need to investigate the behavior of<br />

Fibonacci-type sequences modulo 2 n .<br />

Denote by F 0 , F 1 , . . . the Fibonacci numbers defined by F 00, F 11, <strong>and</strong> F n<br />

F n 1 F n for n0. We also set F¡11 according to the recurrence relation.<br />

νÔmÕ k<br />

For every positive integer m, denote by νÔmÕthe exponent of 2 in the prime factorization<br />

of m, i.e. for which 2 νÔmÕ§m but 2 1Чm.<br />

Lemma 1. For every Fibonacci-type sequence a 0 , a 1 , a 2 , . . ., <strong>and</strong> every k0, we have a<br />

F k¡1a 0 F k a 1 .<br />

Proof. Apply induction on k. The base cases<br />

1Õ Ð<br />

k0, 1 are trivial. For the step, from the<br />

induction hypothesis we get<br />

a k 1a k a k¡1ÔF k¡1a 0 F k a ÔF k¡2a 0 F k¡1a 1ÕF k a 0 F k 1a 1 .<br />

Lemma 2. For every m3,<br />

(a) we have νÔF m¡2Õm;<br />

3¤2 (b) d3¤2m¡2 is the least positive index for which 2 m§F d ;<br />

(c) F 3¤2m¡2 11 2 m¡1Ômod 2 mÕ.<br />

Proof. Apply induction on m. In the base case m3we have νÔF m¡2ÕF 3¤2 68,<br />

so<br />

νÔF m¡2ÕνÔ8Õ3, 3¤2 the preceding Fibonacci-numbers are not divisible by 8, <strong>and</strong> indeed<br />

F 3¤2m¡2 1F 7131 4Ômod 8Õ.<br />

Now suppose that m3 <strong>and</strong> let k3¤2m¡3 . By applying Lemma 1 to the Fibonacci-type<br />

2Ôm¡1ÕÔm¡1Õ<br />

sequence F k , F k 1, . . . we get<br />

F 2kF k¡1F k F k F k 1ÔF k 1¡F kÕF k F k 1F k2F k 1F k¡F kÕ<br />

k 2 ,<br />

F 2k 1F k¤F k F k 1¤F k 1F k 2 Fk 2 1.<br />

By the induction hypothesis, νÔF kÕm¡1, <strong>and</strong> F k 1 is odd. Therefore we get νÔF 2<br />

1νÔ2F k F k 1Õ, which implies νÔF 2kÕm, establishing statement (a).


73<br />

Moreover, since F k 11 2 m¡2 a2 m¡1 for some integer a, we get<br />

F 2k 1F k 2 Fk 2 10 Ô1 2 m¡2 a2 m¡1Õ21 2 m¡1Ômod 2 mÕ,<br />

as desired in statement (c).<br />

We are left to prove that 2 mЧF l for l2k. Assume the contrary. Since 2 m¡1§F l<br />

Ð<br />

, from<br />

the induction hypothesis it follows that lk. But then we have F lF k¡1F l¡k F k F l¡k 1,<br />

where the second summ<strong>and</strong> is divisible by 2 m¡1 but the first one is not (since F k¡1 is odd <strong>and</strong><br />

l¡kk). Hence the sum is not divisible even by 2 m¡1 . A contradiction.<br />

Now, for every pair of integersÔa, bÕÔ0, 0Õ, let µÔa, bÕminØνÔaÕ, νÔbÕÙ. By an obvious induction,<br />

for every Fibonacci-type sequence AÔa 0 , a 1 , . . .Õwe have µÔa 0 , a 1ÕµÔa 1 , a 2Õ. . .;<br />

denote this common value by µÔAÕ. Also denote by p nÔAÕthe period of this sequence modulo<br />

2 n , that is, the least p0 such that a k pa kÔmod 2 nÕfor all k0.<br />

Lemma 3. Let AÔa 0 , a 1 , . . .Õbe a Fibonacci-type sequence such that µÔAÕkn. Then<br />

p nÔAÕ3¤2n¡1¡k .<br />

Proof. First, we note that the sequenceÔa 0 , a 1 , . . .Õhas period p modulo 2 n if <strong>and</strong> only if the<br />

sequenceÔa 0ß2k , a 1ß2k , . . .Õhas period p modulo 2 n¡k . Hence, passing to this sequence we can<br />

.Õ<br />

assume that k0.<br />

We prove the statement by induction on n. It is easy to see that for n1, 2 the claim<br />

is true; actually, each Fibonacci-type sequence A with µÔAÕ0behaves as 0, 1, 1, 0, 1, 1, . . .<br />

modulo 2, <strong>and</strong> as 0, 1, 1, 2, 3, 1, 0, 1, 1, 2, 3, 1, ... modulo 4 (all pairs of residues from which at<br />

least one is odd appear as a pair of consecutive terms in this sequence).<br />

Now suppose that n3<strong>and</strong> consider an arbitrary Fibonacci-type sequence AÔa 0 , a 1 , . .<br />

with µÔAÕ0. Obviously we should have p n¡1ÔAÕ§p nÔAÕ, or, using the induction hypothesis,<br />

0<br />

s3¤2n¡2§p nÔAÕ. Next, we may suppose that a 0 is even; hence a 1 is odd, <strong>and</strong> a 02b 0 ,<br />

a 12b 1 1 for some integers b 0 , b 1 .<br />

Consider the Fibonacci-type sequence BÔb 0 , b 1 , . . .Õstarting withÔb 0 , b 1Õ. Since a<br />

2b 0 F 0 , a 12b 1 F 1 , by an easy induction we get a k2b k F k for all k0. By<br />

the induction hypothesis, we have p n¡1ÔBÕ§s, hence the sequenceÔ2b 0 , 2b 1 , . . .Õis s-periodic<br />

modulo 2 n . On the other h<strong>and</strong>, by Lemma 2 we have F s 11 2 n¡1Ômod 2 nÕ, F 2s0<br />

Ômod 2 nÕ, F 2s 11Ômod 2 nÕ, hence<br />

a s 12b s 1 F s 12b 1 1 2 n¡12b 1 1a 1Ômod 2 nÕ,<br />

a 2s2b 2s F 2s2b 0 0a 0Ômod 2 nÕ,<br />

a 2s 12b 2s 1 F 2s 12b 1 1a 1Ômod<br />

Ð<br />

2 nÕ.<br />

The first line means that A is not s-periodic, while the other two provide that a 2sa 0 ,<br />

a 2s 1a 1 <strong>and</strong> hence a 2s ta t for all t0. Hence s§p nÔAÕ§2s <strong>and</strong> p nÔAÕs, which means<br />

that p nÔAÕ2s, as desired.<br />

0Õ<br />

Finally, Lemma 3 provides a straightforward method of counting the number of cycles.<br />

Actually, take any number 0kn¡1 <strong>and</strong> consider all the cellsÔi, jÕwith µÔi, jÕk. The<br />

total number of such cells is 22Ôn¡kÕ¡22Ôn¡k¡1Õ3¤22n¡2k¡2 . On the other h<strong>and</strong>, they are split<br />

into cycles, <strong>and</strong> by Lemma 3 the length of each cycle is<br />

¤¤¤<br />

3¤2n¡1¡k . Hence the number of cycles<br />

2n¡2k¡2<br />

3¤2 consisting of these cells is exactly<br />

3¤2n¡1¡k2 n¡k¡1 . Finally, there is only one cellÔ0,<br />

which is not mentioned in the previous computation, <strong>and</strong> it forms a separate cycle. So the total<br />

number of cycles is<br />

n¡1<br />

1 2<br />

ôk0<br />

n¡1¡k1 Ô1 2 4 2 n¡1Õ2 n .


74<br />

Comment. We outline a different proof for the essential part of Lemma 3. That is, we assume that<br />

k0<strong>and</strong> show that in this case the period ofÔa iÕmodulo 2 n coincides with the period of the Fibonacci<br />

numbers modulo 2 n ; then the proof can be finished by the arguments from Lemma 2..<br />

Note that p is a (not necessarily minimal) period of the<br />

0Õ<br />

sequenceÔa iÕmodulo 2 n if <strong>and</strong> only if we<br />

have a 0a pÔmod 2 nÕ, a 1a p 1Ômod 2 nÕ, that is,<br />

a 0a pF p¡1a 0 F p a 1F pÔa 1¡a F p 1a 0Ômod 2 nÕ,<br />

a 1a p 1F p a 0 F p 1a 1Ômod 2 nÕ.<br />

(1)<br />

Now, If p is a period ofÔF iÕthen we have F pF 00Ômod 2 nÕ<strong>and</strong> F p 1F 11Ômod 2 nÕ, which<br />

by (1) implies that p is a period ofÔa iÕas well.<br />

0Õ<br />

Conversely, suppose that p is a period ofÔa iÕ. Combining the relations of (1) we<br />

0Õ 1Õ<br />

get<br />

0¨<br />

0a 1¤a 0¡a 0¤a 1a 1 F pÔa 1¡a F p 1a 0¨¡a 0ÔF p a 0 F p 1a<br />

F pÔa 2 1¡a 1 a 0¡a 2 0ÕÔmod 2 nÕ,<br />

a 2 1¡a 1 a 0¡a 2 0Ôa 1¡a 0Õa 1¡a 0¤a 0Ôa 1¡a 0ÕÔF p a 0 F p 1a 1Õ¡a 0 F pÔa 1¡a F p 1a<br />

F p 1Ôa 2 1¡a 1 a 0¡a 2 0ÕÔmod 2 nÕ.<br />

Since at least one of the numbers a 0 , a 1 is odd, the number a 2 1¡a 1 a 0¡a2 0 is odd as well. Therefore the<br />

previous relations are equivalent with F p0Ômod 2 nÕ<strong>and</strong> F p 11Ômod 2 nÕ, which means exactly<br />

that p is a period ofÔF 0 ,F 1 ,...Õmodulo 2 n .<br />

So, the sets of periods ofÔa iÕ<strong>and</strong>ÔF iÕcoincide, <strong>and</strong> hence the minimal periods coincide as well.

Hooray! Your file is uploaded and ready to be published.

Saved successfully!

Ooh no, something went wrong!